How should you treat the newly diagnosed hypertensive patient?

Article Type
Changed
Mon, 01/14/2019 - 11:29
Display Headline
How should you treat the newly diagnosed hypertensive patient?
EVIDENCE-BASED ANSWER

IT DEPENDS ON THE PATIENT’S RISK FACTORS, physical condition, and preferences. All hypertensive patients can potentially benefit from lifestyle interventions, including weight reduction, aerobic physical activity, the dietary approaches to stop hypertension (DASH) diet, and moderation of alcohol use (strength of recommendation [SOR]: A, systematic reviews).

Although lifestyle interventions are effective for some patients, they haven’t been proven to provide long-term control and don’t lower blood pressure as much as medications (SOR: B, systematic review of inconsistent randomized controlled trial [RCT]). For specific high-risk patients, pharmacologic therapy is recommended at the time of diagnosis (SOR: C, expert opinion).

When considering lifestyle changes and medication, it’s important to assess patient preferences as well as overall cardiovascular risks, presence of target organ damage, and clinical cardiovascular disease, because lifestyle modification and medication can both affect quality of life (SOR: C, expert opinion).

 

Evidence summary

The prevalence of hypertension is increasing. Twenty-seven percent of adult Americans are hypertensive; 31% have prehypertension (TABLE).1 Among adults older than 50 years, the risk of developing high blood pressure approaches 90% if they live to age 80 or older.2 Cardiovascular risk rises along with blood pressure readings. Blood pressure values in the range of 130/85 to 139/89 mm Hg are associated with a more than 2-fold increase in cardiovascular disease risk compared with values below 120/80 mm Hg.2

Even small reductions in blood pressure, when applied to the population as a whole, produce significant improvements in patient-oriented outcomes. A drop in systolic blood pressure of 3 mm Hg can decrease stroke mortality by 8% and coronary artery disease by 5%.2

Lifestyle interventions for treating hypertension
Lifestyle interventions for hypertensive patients include:

  • Striving to maintain or achieve an ideal body weight (body mass index of 18-25).2 However, even modest weight reductions in overweight and obese hypertensive patients significantly lower blood pressure and overall cardiovascular risk. A 10-kg decrease in body weight can lower systolic blood pressure by 5 to 20 mm Hg.2
  • Adopting the DASH diet.1 Consuming a diet rich in fruits, vegetables, and lowfat dairy products and limiting saturated and total fat intake can reduce systolic blood pressure by 2 to 8 mm Hg.
  • Engaging in regular physical activity for at least 30 minutes most days of the week. This regimen has been shown to decrease systolic blood pressure by 4 to 9 mm Hg.2
  • Limiting daily alcohol intake, if the patient drinks, to 2 servings for men and 1 for women and lower-weight individuals.2 Restricting alcohol consumption may lower systolic blood pressure by 2 to 4 mm Hg.

Randomized controlled trials have consistently demonstrated that patients who combine multiple lifestyle interventions achieve the greatest benefits.3-5 Success obviously requires patient motivation. Health care providers need to continually assess motivation and encourage adherence.

Most patients need medication, too
Lifestyle changes alone haven’t been shown to achieve the same long-term reductions in blood pressure as medication.5 Although some motivated patients can control their blood pressure solely by adjusting their lifestyle, few succeed in reaching and maintaining blood pressure goals. Continued attention to lifestyle should be encouraged both to control blood pressure and reduce overall cardiovascular risk, but most patients with hypertension need medication.

The Seventh Report of the Joint National Committee on Prevention, Detection, Evaluation, and Treatment of High Blood Pressure (JNC7) doesn’t specify whether to offer a trial of therapeutic lifestyle change before starting medication.2 Clinicians should negotiate interventions based on each patient’s preferences, risk factors, and the presence or absence of clinical cardiovascular disease or target organ damage. Lifestyle changes and medication both can affect quality of life. Immediate pharmacologic treatment with 2 medications has been recommended in addition to lifestyle interventions for patients with stage 2 hypertension (TABLE).2

TABLE
JNC7 classifications of blood pressure in adults

Blood pressure classificationBlood pressure (mm Hg)Recommended follow-up
NormalSystolic <120
AND
diastolic <80
2 y
PrehypertensionSystolic 120-139
OR
diastolic 80-89
1 y
Stage 1 hypertensionSystolic 140-159
OR
diastolic 90-99
2 mo
Stage 2 hypertensionSystolic ≥160
OR
diastolic ≥100
1 mo*
JNC7, Seventh Report of the Joint National Committee on Prevention, Detection, Evaluation, and Treatment of High Blood Pressure.
*For people with higher values (>180/110 mm Hg), evaluate and treat immediately or within 1 week, depending on clinical situation.
Source: Chobanian AV, et al. Hypertension. 2003.2
 

 

 

Recommendations

The European Society of Hypertension provides recommendations for the duration of lifestyle interventions before trying medication. The recommendations are based on a complex scheme of overall cardiovascular risk assessment that takes into account traditional Framingham risks and other factors (such as obesity, C-reactive protein, and micro albuminuria), as well as the stage of hypertension.6 The Society recommends starting drug therapy immediately in people with blood pressure >180/110 mm Hg. This blood pressure threshold drops in patients with increasing numbers of risk factors. For patients with lower, but still elevated, blood pressure, the recommendations call for “lifestyle changes for several months, then drug treatment if BP is uncontrolled.”

For patients with diabetes, the American Diabetes Association (ADA) recommends a blood pressure goal of <130/80 mm Hg and drug therapy in addition to lifestyle and behavioral therapy for patients with systolic blood pressure ≥140 mm Hg or diastolic blood pressure ≥90 mm Hg.7 Like the JNC7, the ADA notes that a combination of medications is often required to achieve blood pressure targets. The ADA recommendations also state that patients with diabetes and a systolic blood pressure of 130 to 139 mm Hg or a diastolic blood pressure of 80 to 89 mm Hg should pursue lifestyle and behavioral interventions alone for a maximum of 3 months, then start drug therapy if they don’t achieve their blood pressure goals.

The American Heart Association and American College of Cardiology offer evidence-based guidelines for secondary prevention in patients with atherosclerosis.8 They set blood pressure goals of <140/90 mm Hg for all patients and <130/80 mm Hg for patients with diabetes or chronic kidney disease. All patients are encouraged to initiate or maintain lifestyle modifications. If a patient’s blood pressure is ≥140/90 mm Hg (>130/80 mm Hg for patients with chronic kidney disease or diabetes), medications should be titrated to goal, beginning with beta-blockers or angiotensin-converting enzyme inhibitors.

References

1. Appel LJ, Brands MW, Daniels SR, et al. Dietary approaches to prevent and treat hypertension: a scientific statement from the American Heart Association. Hypertension. 2006;47:296-308.

2. Chobanian AV, Bakris GL, Black HR, et al. Seventh report of the Joint National Committee on Prevention, Detection, Evaluation, and Treatment of High Blood Pressure. Hypertension. 2003;42:1206-1252.

3. Burke V, Beilin LJ, Cutt HE, et al. Effects of a lifestyle programme on ambulatory blood pressure and drug dosage in treated hypertensive patients: a randomized controlled trial. J Hypertens. 2005;23:1241-1249.

4. Elmer PJ, Obarzanek E, Vollmer WM, et al. Effects of comprehensive lifestyle modification on diet, weight, physical fitness, and blood pressure control: 18-month results of a randomized trial. Ann Intern Med. 2006;144:485-495.

5. Nicolson DJ, Dickinson HO, Campbell F, et al. Lifestyle interventions or drugs for patients with essential hypertension: a systematic review. J Hypertens. 2004;22:2043-2048.

6. European Society of Hypertension-European Society of Cardiology Task Force on the Management of Arterial Hypertension. 2007 ESH-ESC practice guidelines for the management of arterial hypertension. J Hypertens. 2007;25:1751-1762.

7. American Diabetes Association. Standards of medical care in diabetes 2010. Diabetes Care. 2010;33(suppl 1):S11-S61.Available at: http://care.diabetesjournals.org/content/33/Supplement_1. Accessed March 1, 2010.

8. American Hospital Association, American College of Cardiology. AHA-ACC guidelines for secondary prevention for patients with coronary and other atherosclerotic vascular disease: 2006 update. Circulation. 2006;113:2363-2372.

Article PDF
Author and Disclosure Information

Gary Debrino, MD
Michael Grover, DO
Department of Family Medicine, Mayo Clinic, Scottsdale, Ariz

William Nichols, MLS
Eglin Air Force Base, Fla

Issue
The Journal of Family Practice - 59(11)
Publications
Topics
Page Number
651-653
Legacy Keywords
Gary Debrino;MD; lifestyle interventions; pharmacologic therapy; cardiovascular risks; dietary; prehypertension; coronary artery disease
Sections
Author and Disclosure Information

Gary Debrino, MD
Michael Grover, DO
Department of Family Medicine, Mayo Clinic, Scottsdale, Ariz

William Nichols, MLS
Eglin Air Force Base, Fla

Author and Disclosure Information

Gary Debrino, MD
Michael Grover, DO
Department of Family Medicine, Mayo Clinic, Scottsdale, Ariz

William Nichols, MLS
Eglin Air Force Base, Fla

Article PDF
Article PDF
EVIDENCE-BASED ANSWER

IT DEPENDS ON THE PATIENT’S RISK FACTORS, physical condition, and preferences. All hypertensive patients can potentially benefit from lifestyle interventions, including weight reduction, aerobic physical activity, the dietary approaches to stop hypertension (DASH) diet, and moderation of alcohol use (strength of recommendation [SOR]: A, systematic reviews).

Although lifestyle interventions are effective for some patients, they haven’t been proven to provide long-term control and don’t lower blood pressure as much as medications (SOR: B, systematic review of inconsistent randomized controlled trial [RCT]). For specific high-risk patients, pharmacologic therapy is recommended at the time of diagnosis (SOR: C, expert opinion).

When considering lifestyle changes and medication, it’s important to assess patient preferences as well as overall cardiovascular risks, presence of target organ damage, and clinical cardiovascular disease, because lifestyle modification and medication can both affect quality of life (SOR: C, expert opinion).

 

Evidence summary

The prevalence of hypertension is increasing. Twenty-seven percent of adult Americans are hypertensive; 31% have prehypertension (TABLE).1 Among adults older than 50 years, the risk of developing high blood pressure approaches 90% if they live to age 80 or older.2 Cardiovascular risk rises along with blood pressure readings. Blood pressure values in the range of 130/85 to 139/89 mm Hg are associated with a more than 2-fold increase in cardiovascular disease risk compared with values below 120/80 mm Hg.2

Even small reductions in blood pressure, when applied to the population as a whole, produce significant improvements in patient-oriented outcomes. A drop in systolic blood pressure of 3 mm Hg can decrease stroke mortality by 8% and coronary artery disease by 5%.2

Lifestyle interventions for treating hypertension
Lifestyle interventions for hypertensive patients include:

  • Striving to maintain or achieve an ideal body weight (body mass index of 18-25).2 However, even modest weight reductions in overweight and obese hypertensive patients significantly lower blood pressure and overall cardiovascular risk. A 10-kg decrease in body weight can lower systolic blood pressure by 5 to 20 mm Hg.2
  • Adopting the DASH diet.1 Consuming a diet rich in fruits, vegetables, and lowfat dairy products and limiting saturated and total fat intake can reduce systolic blood pressure by 2 to 8 mm Hg.
  • Engaging in regular physical activity for at least 30 minutes most days of the week. This regimen has been shown to decrease systolic blood pressure by 4 to 9 mm Hg.2
  • Limiting daily alcohol intake, if the patient drinks, to 2 servings for men and 1 for women and lower-weight individuals.2 Restricting alcohol consumption may lower systolic blood pressure by 2 to 4 mm Hg.

Randomized controlled trials have consistently demonstrated that patients who combine multiple lifestyle interventions achieve the greatest benefits.3-5 Success obviously requires patient motivation. Health care providers need to continually assess motivation and encourage adherence.

Most patients need medication, too
Lifestyle changes alone haven’t been shown to achieve the same long-term reductions in blood pressure as medication.5 Although some motivated patients can control their blood pressure solely by adjusting their lifestyle, few succeed in reaching and maintaining blood pressure goals. Continued attention to lifestyle should be encouraged both to control blood pressure and reduce overall cardiovascular risk, but most patients with hypertension need medication.

The Seventh Report of the Joint National Committee on Prevention, Detection, Evaluation, and Treatment of High Blood Pressure (JNC7) doesn’t specify whether to offer a trial of therapeutic lifestyle change before starting medication.2 Clinicians should negotiate interventions based on each patient’s preferences, risk factors, and the presence or absence of clinical cardiovascular disease or target organ damage. Lifestyle changes and medication both can affect quality of life. Immediate pharmacologic treatment with 2 medications has been recommended in addition to lifestyle interventions for patients with stage 2 hypertension (TABLE).2

TABLE
JNC7 classifications of blood pressure in adults

Blood pressure classificationBlood pressure (mm Hg)Recommended follow-up
NormalSystolic <120
AND
diastolic <80
2 y
PrehypertensionSystolic 120-139
OR
diastolic 80-89
1 y
Stage 1 hypertensionSystolic 140-159
OR
diastolic 90-99
2 mo
Stage 2 hypertensionSystolic ≥160
OR
diastolic ≥100
1 mo*
JNC7, Seventh Report of the Joint National Committee on Prevention, Detection, Evaluation, and Treatment of High Blood Pressure.
*For people with higher values (>180/110 mm Hg), evaluate and treat immediately or within 1 week, depending on clinical situation.
Source: Chobanian AV, et al. Hypertension. 2003.2
 

 

 

Recommendations

The European Society of Hypertension provides recommendations for the duration of lifestyle interventions before trying medication. The recommendations are based on a complex scheme of overall cardiovascular risk assessment that takes into account traditional Framingham risks and other factors (such as obesity, C-reactive protein, and micro albuminuria), as well as the stage of hypertension.6 The Society recommends starting drug therapy immediately in people with blood pressure >180/110 mm Hg. This blood pressure threshold drops in patients with increasing numbers of risk factors. For patients with lower, but still elevated, blood pressure, the recommendations call for “lifestyle changes for several months, then drug treatment if BP is uncontrolled.”

For patients with diabetes, the American Diabetes Association (ADA) recommends a blood pressure goal of <130/80 mm Hg and drug therapy in addition to lifestyle and behavioral therapy for patients with systolic blood pressure ≥140 mm Hg or diastolic blood pressure ≥90 mm Hg.7 Like the JNC7, the ADA notes that a combination of medications is often required to achieve blood pressure targets. The ADA recommendations also state that patients with diabetes and a systolic blood pressure of 130 to 139 mm Hg or a diastolic blood pressure of 80 to 89 mm Hg should pursue lifestyle and behavioral interventions alone for a maximum of 3 months, then start drug therapy if they don’t achieve their blood pressure goals.

The American Heart Association and American College of Cardiology offer evidence-based guidelines for secondary prevention in patients with atherosclerosis.8 They set blood pressure goals of <140/90 mm Hg for all patients and <130/80 mm Hg for patients with diabetes or chronic kidney disease. All patients are encouraged to initiate or maintain lifestyle modifications. If a patient’s blood pressure is ≥140/90 mm Hg (>130/80 mm Hg for patients with chronic kidney disease or diabetes), medications should be titrated to goal, beginning with beta-blockers or angiotensin-converting enzyme inhibitors.

EVIDENCE-BASED ANSWER

IT DEPENDS ON THE PATIENT’S RISK FACTORS, physical condition, and preferences. All hypertensive patients can potentially benefit from lifestyle interventions, including weight reduction, aerobic physical activity, the dietary approaches to stop hypertension (DASH) diet, and moderation of alcohol use (strength of recommendation [SOR]: A, systematic reviews).

Although lifestyle interventions are effective for some patients, they haven’t been proven to provide long-term control and don’t lower blood pressure as much as medications (SOR: B, systematic review of inconsistent randomized controlled trial [RCT]). For specific high-risk patients, pharmacologic therapy is recommended at the time of diagnosis (SOR: C, expert opinion).

When considering lifestyle changes and medication, it’s important to assess patient preferences as well as overall cardiovascular risks, presence of target organ damage, and clinical cardiovascular disease, because lifestyle modification and medication can both affect quality of life (SOR: C, expert opinion).

 

Evidence summary

The prevalence of hypertension is increasing. Twenty-seven percent of adult Americans are hypertensive; 31% have prehypertension (TABLE).1 Among adults older than 50 years, the risk of developing high blood pressure approaches 90% if they live to age 80 or older.2 Cardiovascular risk rises along with blood pressure readings. Blood pressure values in the range of 130/85 to 139/89 mm Hg are associated with a more than 2-fold increase in cardiovascular disease risk compared with values below 120/80 mm Hg.2

Even small reductions in blood pressure, when applied to the population as a whole, produce significant improvements in patient-oriented outcomes. A drop in systolic blood pressure of 3 mm Hg can decrease stroke mortality by 8% and coronary artery disease by 5%.2

Lifestyle interventions for treating hypertension
Lifestyle interventions for hypertensive patients include:

  • Striving to maintain or achieve an ideal body weight (body mass index of 18-25).2 However, even modest weight reductions in overweight and obese hypertensive patients significantly lower blood pressure and overall cardiovascular risk. A 10-kg decrease in body weight can lower systolic blood pressure by 5 to 20 mm Hg.2
  • Adopting the DASH diet.1 Consuming a diet rich in fruits, vegetables, and lowfat dairy products and limiting saturated and total fat intake can reduce systolic blood pressure by 2 to 8 mm Hg.
  • Engaging in regular physical activity for at least 30 minutes most days of the week. This regimen has been shown to decrease systolic blood pressure by 4 to 9 mm Hg.2
  • Limiting daily alcohol intake, if the patient drinks, to 2 servings for men and 1 for women and lower-weight individuals.2 Restricting alcohol consumption may lower systolic blood pressure by 2 to 4 mm Hg.

Randomized controlled trials have consistently demonstrated that patients who combine multiple lifestyle interventions achieve the greatest benefits.3-5 Success obviously requires patient motivation. Health care providers need to continually assess motivation and encourage adherence.

Most patients need medication, too
Lifestyle changes alone haven’t been shown to achieve the same long-term reductions in blood pressure as medication.5 Although some motivated patients can control their blood pressure solely by adjusting their lifestyle, few succeed in reaching and maintaining blood pressure goals. Continued attention to lifestyle should be encouraged both to control blood pressure and reduce overall cardiovascular risk, but most patients with hypertension need medication.

The Seventh Report of the Joint National Committee on Prevention, Detection, Evaluation, and Treatment of High Blood Pressure (JNC7) doesn’t specify whether to offer a trial of therapeutic lifestyle change before starting medication.2 Clinicians should negotiate interventions based on each patient’s preferences, risk factors, and the presence or absence of clinical cardiovascular disease or target organ damage. Lifestyle changes and medication both can affect quality of life. Immediate pharmacologic treatment with 2 medications has been recommended in addition to lifestyle interventions for patients with stage 2 hypertension (TABLE).2

TABLE
JNC7 classifications of blood pressure in adults

Blood pressure classificationBlood pressure (mm Hg)Recommended follow-up
NormalSystolic <120
AND
diastolic <80
2 y
PrehypertensionSystolic 120-139
OR
diastolic 80-89
1 y
Stage 1 hypertensionSystolic 140-159
OR
diastolic 90-99
2 mo
Stage 2 hypertensionSystolic ≥160
OR
diastolic ≥100
1 mo*
JNC7, Seventh Report of the Joint National Committee on Prevention, Detection, Evaluation, and Treatment of High Blood Pressure.
*For people with higher values (>180/110 mm Hg), evaluate and treat immediately or within 1 week, depending on clinical situation.
Source: Chobanian AV, et al. Hypertension. 2003.2
 

 

 

Recommendations

The European Society of Hypertension provides recommendations for the duration of lifestyle interventions before trying medication. The recommendations are based on a complex scheme of overall cardiovascular risk assessment that takes into account traditional Framingham risks and other factors (such as obesity, C-reactive protein, and micro albuminuria), as well as the stage of hypertension.6 The Society recommends starting drug therapy immediately in people with blood pressure >180/110 mm Hg. This blood pressure threshold drops in patients with increasing numbers of risk factors. For patients with lower, but still elevated, blood pressure, the recommendations call for “lifestyle changes for several months, then drug treatment if BP is uncontrolled.”

For patients with diabetes, the American Diabetes Association (ADA) recommends a blood pressure goal of <130/80 mm Hg and drug therapy in addition to lifestyle and behavioral therapy for patients with systolic blood pressure ≥140 mm Hg or diastolic blood pressure ≥90 mm Hg.7 Like the JNC7, the ADA notes that a combination of medications is often required to achieve blood pressure targets. The ADA recommendations also state that patients with diabetes and a systolic blood pressure of 130 to 139 mm Hg or a diastolic blood pressure of 80 to 89 mm Hg should pursue lifestyle and behavioral interventions alone for a maximum of 3 months, then start drug therapy if they don’t achieve their blood pressure goals.

The American Heart Association and American College of Cardiology offer evidence-based guidelines for secondary prevention in patients with atherosclerosis.8 They set blood pressure goals of <140/90 mm Hg for all patients and <130/80 mm Hg for patients with diabetes or chronic kidney disease. All patients are encouraged to initiate or maintain lifestyle modifications. If a patient’s blood pressure is ≥140/90 mm Hg (>130/80 mm Hg for patients with chronic kidney disease or diabetes), medications should be titrated to goal, beginning with beta-blockers or angiotensin-converting enzyme inhibitors.

References

1. Appel LJ, Brands MW, Daniels SR, et al. Dietary approaches to prevent and treat hypertension: a scientific statement from the American Heart Association. Hypertension. 2006;47:296-308.

2. Chobanian AV, Bakris GL, Black HR, et al. Seventh report of the Joint National Committee on Prevention, Detection, Evaluation, and Treatment of High Blood Pressure. Hypertension. 2003;42:1206-1252.

3. Burke V, Beilin LJ, Cutt HE, et al. Effects of a lifestyle programme on ambulatory blood pressure and drug dosage in treated hypertensive patients: a randomized controlled trial. J Hypertens. 2005;23:1241-1249.

4. Elmer PJ, Obarzanek E, Vollmer WM, et al. Effects of comprehensive lifestyle modification on diet, weight, physical fitness, and blood pressure control: 18-month results of a randomized trial. Ann Intern Med. 2006;144:485-495.

5. Nicolson DJ, Dickinson HO, Campbell F, et al. Lifestyle interventions or drugs for patients with essential hypertension: a systematic review. J Hypertens. 2004;22:2043-2048.

6. European Society of Hypertension-European Society of Cardiology Task Force on the Management of Arterial Hypertension. 2007 ESH-ESC practice guidelines for the management of arterial hypertension. J Hypertens. 2007;25:1751-1762.

7. American Diabetes Association. Standards of medical care in diabetes 2010. Diabetes Care. 2010;33(suppl 1):S11-S61.Available at: http://care.diabetesjournals.org/content/33/Supplement_1. Accessed March 1, 2010.

8. American Hospital Association, American College of Cardiology. AHA-ACC guidelines for secondary prevention for patients with coronary and other atherosclerotic vascular disease: 2006 update. Circulation. 2006;113:2363-2372.

References

1. Appel LJ, Brands MW, Daniels SR, et al. Dietary approaches to prevent and treat hypertension: a scientific statement from the American Heart Association. Hypertension. 2006;47:296-308.

2. Chobanian AV, Bakris GL, Black HR, et al. Seventh report of the Joint National Committee on Prevention, Detection, Evaluation, and Treatment of High Blood Pressure. Hypertension. 2003;42:1206-1252.

3. Burke V, Beilin LJ, Cutt HE, et al. Effects of a lifestyle programme on ambulatory blood pressure and drug dosage in treated hypertensive patients: a randomized controlled trial. J Hypertens. 2005;23:1241-1249.

4. Elmer PJ, Obarzanek E, Vollmer WM, et al. Effects of comprehensive lifestyle modification on diet, weight, physical fitness, and blood pressure control: 18-month results of a randomized trial. Ann Intern Med. 2006;144:485-495.

5. Nicolson DJ, Dickinson HO, Campbell F, et al. Lifestyle interventions or drugs for patients with essential hypertension: a systematic review. J Hypertens. 2004;22:2043-2048.

6. European Society of Hypertension-European Society of Cardiology Task Force on the Management of Arterial Hypertension. 2007 ESH-ESC practice guidelines for the management of arterial hypertension. J Hypertens. 2007;25:1751-1762.

7. American Diabetes Association. Standards of medical care in diabetes 2010. Diabetes Care. 2010;33(suppl 1):S11-S61.Available at: http://care.diabetesjournals.org/content/33/Supplement_1. Accessed March 1, 2010.

8. American Hospital Association, American College of Cardiology. AHA-ACC guidelines for secondary prevention for patients with coronary and other atherosclerotic vascular disease: 2006 update. Circulation. 2006;113:2363-2372.

Issue
The Journal of Family Practice - 59(11)
Issue
The Journal of Family Practice - 59(11)
Page Number
651-653
Page Number
651-653
Publications
Publications
Topics
Article Type
Display Headline
How should you treat the newly diagnosed hypertensive patient?
Display Headline
How should you treat the newly diagnosed hypertensive patient?
Legacy Keywords
Gary Debrino;MD; lifestyle interventions; pharmacologic therapy; cardiovascular risks; dietary; prehypertension; coronary artery disease
Legacy Keywords
Gary Debrino;MD; lifestyle interventions; pharmacologic therapy; cardiovascular risks; dietary; prehypertension; coronary artery disease
Sections
PURLs Copyright

Evidence-based answers from the Family Physicians Inquiries Network

Disallow All Ads
Article PDF Media

Does heat or cold work better for acute muscle strain?

Article Type
Changed
Mon, 01/14/2019 - 11:24
Display Headline
Does heat or cold work better for acute muscle strain?
EVIDENCE-BASED ANSWER

Cryotherapy is better than heat for treating acute muscle strain (strength of recommendation [SOR]: C, consensus, usual practice, and expert opinion). Insufficient patient-oriented evidence exists regarding use of heat to treat acute soft-tissue injuries.

 

Evidence summary

A comprehensive review of the literature revealed no studies that compare heat and cryotherapy to treat acute soft-tissue injury. Well-designed human trials of general management of acute soft-tissue injury are rare.1

Cryotherapy has been the recommended initial treatment for muscle strain for more than 30 years, based generally on expert opinion and physiological models, not clinical trials.2 Theoretically, cryotherapy controls hemorrhage and tissue edema, whereas heat enhances the inflammatory response.2

One human RCT and animal studies find benefits from cold

A 2007 review evaluated 66 publications and found only 1 randomized controlled trial conducted on humans.3 The intervention in this trial involved applying cold gel 4 times a day for the first 14 days after the injury. The control group received a room-temperature gel application; neither group was aware of the temperature differential.

The study found significant reduction in pain at rest, pain with movement, and functional disability at intervals of 7, 14, and 28 days postinjury (P<.001) among patients receiving cold-gel applications. Patients receiving cold-gel treatment also reported increased satisfaction with treatment compared with the controls. At 28 days, cold-gel treatment patients scored 71 on a 100-point satisfaction scale compared with 44 for controls (P<.001).3 Inconclusive results or significant design flaws limited the validity of all other trials cited in this review.3

Laboratory studies on rats have also demonstrated beneficial effects of cryotherapy after simulated soft-tissue injuries.4,5 One study cited a significant reduction in inflammatory cells, based on histologic examination, in 43 rats between 6 and 24 hours after trauma.4 A second study of 21 rats showed improvement in associated physiological components with cryotherapy, but no statistically significant improvement in edema.5

How cold is too cold?

Most authorities recommend empiric treatment with cryotherapy during the acute inflammatory phase—the first 24 to 48 hours after injury.6 Although not rigorously studied, some sources recommend applying cold to the involved muscle for the first 4 hours after injury at intervals of 10 to 20 minutes every 30 to 60 minutes.6

The literature focuses more on the optimal temperature for cryotherapy than on the duration and frequency of therapy.7 Temperatures below 15°to 25°C may actually result in vasodilatation rather than vasoconstriction.7

 

 

 

Evidence for heat is limited

A 2006 Cochrane review that addressed treatment of lower back muscular strain, not soft-tissue injuries in general, found moderate evidence that heat therapy reduces pain by 17% and disability in the acute setting (P=.001).8 The review also cited 2 head-to-head trials that compared heat and cryotherapy; however, the study designs were poor and the results were contradictory.8

Recommendations

Authoritative textbooks consistently recommend applying ice for initial treatment of musculoskeletal and soft-tissue strains.9

Acknowledgments

The opinions and assertions contained herein are the private views of the authors and not to be construed as official or as reflecting the views of the United States Air Force Medical Service or the United States Air Force at large.

References

1. Hubbard TJ, Denegar CR. Does cryotherapy improve outcomes with soft tissue injury? J Athl Train. 2004;39:278-279.

2. Kalenak A, Medlar CE, Fleagle SB, Hochberg WJ. Athletic injuries: heat vs cold. Am Fam Physician. 1975;12:131-134.

3. Collins NC. Is ice right? Does cryotherapy improve outcome for acute soft tissue injury? Emerg Med J. 2008;25:65-68.

4. Hurme T, Rantanen J, Kalimo H. Effects of early cryotherapy in experimental skeletal muscle injury. Scand J Med Sci Sports. 1993;3:46-51.

5. Schaser K, Disch AC, Stover JF, et al. Prolonged superficial local cryotherapy attenuates microcirculatory impairment, regional inflammation, and muscle necrosis after closed soft tissue injury in rats. Am J Sports Med. 2007;35:93-102.

6. Kellett J. Acute soft tissue injuries—a review of the literature. Med Sci Sports Exerc. 1986;18:489-500.

7. McMaster WC, Liddle S, Waugh TR. Laboratory evaluation of various cold therapy modalities. Am J Sports Med. 1978;6:291-294.

8. French SD, Cameron M, Walker BF, Reggars JW, Esterman AJ. A Cochrane review of superficial heat or cold for low back pain. Spine. 2006;31:998-1006.

9. Griffin LY. Essentials of Musculoskeletal Care. 3rd ed. Rosemont, Ill: American Academy of Orthopaedic Surgeons; 2005:134.

Article PDF
Author and Disclosure Information

Sean N. Martin, DO
Christopher P. Paulson, MD
Eglin Air Force Base Family Medicine Residency, Eglin Air Force Base, Fla

William Nichols, MLS
Eglin Air Force Base, Fla

Issue
The Journal of Family Practice - 57(12)
Publications
Page Number
920-921
Legacy Keywords
Dean N. Martin; soft-tissue injury; cryotherapy; gel application; acute muscle strain
Sections
Author and Disclosure Information

Sean N. Martin, DO
Christopher P. Paulson, MD
Eglin Air Force Base Family Medicine Residency, Eglin Air Force Base, Fla

William Nichols, MLS
Eglin Air Force Base, Fla

Author and Disclosure Information

Sean N. Martin, DO
Christopher P. Paulson, MD
Eglin Air Force Base Family Medicine Residency, Eglin Air Force Base, Fla

William Nichols, MLS
Eglin Air Force Base, Fla

Article PDF
Article PDF
EVIDENCE-BASED ANSWER

Cryotherapy is better than heat for treating acute muscle strain (strength of recommendation [SOR]: C, consensus, usual practice, and expert opinion). Insufficient patient-oriented evidence exists regarding use of heat to treat acute soft-tissue injuries.

 

Evidence summary

A comprehensive review of the literature revealed no studies that compare heat and cryotherapy to treat acute soft-tissue injury. Well-designed human trials of general management of acute soft-tissue injury are rare.1

Cryotherapy has been the recommended initial treatment for muscle strain for more than 30 years, based generally on expert opinion and physiological models, not clinical trials.2 Theoretically, cryotherapy controls hemorrhage and tissue edema, whereas heat enhances the inflammatory response.2

One human RCT and animal studies find benefits from cold

A 2007 review evaluated 66 publications and found only 1 randomized controlled trial conducted on humans.3 The intervention in this trial involved applying cold gel 4 times a day for the first 14 days after the injury. The control group received a room-temperature gel application; neither group was aware of the temperature differential.

The study found significant reduction in pain at rest, pain with movement, and functional disability at intervals of 7, 14, and 28 days postinjury (P<.001) among patients receiving cold-gel applications. Patients receiving cold-gel treatment also reported increased satisfaction with treatment compared with the controls. At 28 days, cold-gel treatment patients scored 71 on a 100-point satisfaction scale compared with 44 for controls (P<.001).3 Inconclusive results or significant design flaws limited the validity of all other trials cited in this review.3

Laboratory studies on rats have also demonstrated beneficial effects of cryotherapy after simulated soft-tissue injuries.4,5 One study cited a significant reduction in inflammatory cells, based on histologic examination, in 43 rats between 6 and 24 hours after trauma.4 A second study of 21 rats showed improvement in associated physiological components with cryotherapy, but no statistically significant improvement in edema.5

How cold is too cold?

Most authorities recommend empiric treatment with cryotherapy during the acute inflammatory phase—the first 24 to 48 hours after injury.6 Although not rigorously studied, some sources recommend applying cold to the involved muscle for the first 4 hours after injury at intervals of 10 to 20 minutes every 30 to 60 minutes.6

The literature focuses more on the optimal temperature for cryotherapy than on the duration and frequency of therapy.7 Temperatures below 15°to 25°C may actually result in vasodilatation rather than vasoconstriction.7

 

 

 

Evidence for heat is limited

A 2006 Cochrane review that addressed treatment of lower back muscular strain, not soft-tissue injuries in general, found moderate evidence that heat therapy reduces pain by 17% and disability in the acute setting (P=.001).8 The review also cited 2 head-to-head trials that compared heat and cryotherapy; however, the study designs were poor and the results were contradictory.8

Recommendations

Authoritative textbooks consistently recommend applying ice for initial treatment of musculoskeletal and soft-tissue strains.9

Acknowledgments

The opinions and assertions contained herein are the private views of the authors and not to be construed as official or as reflecting the views of the United States Air Force Medical Service or the United States Air Force at large.

EVIDENCE-BASED ANSWER

Cryotherapy is better than heat for treating acute muscle strain (strength of recommendation [SOR]: C, consensus, usual practice, and expert opinion). Insufficient patient-oriented evidence exists regarding use of heat to treat acute soft-tissue injuries.

 

Evidence summary

A comprehensive review of the literature revealed no studies that compare heat and cryotherapy to treat acute soft-tissue injury. Well-designed human trials of general management of acute soft-tissue injury are rare.1

Cryotherapy has been the recommended initial treatment for muscle strain for more than 30 years, based generally on expert opinion and physiological models, not clinical trials.2 Theoretically, cryotherapy controls hemorrhage and tissue edema, whereas heat enhances the inflammatory response.2

One human RCT and animal studies find benefits from cold

A 2007 review evaluated 66 publications and found only 1 randomized controlled trial conducted on humans.3 The intervention in this trial involved applying cold gel 4 times a day for the first 14 days after the injury. The control group received a room-temperature gel application; neither group was aware of the temperature differential.

The study found significant reduction in pain at rest, pain with movement, and functional disability at intervals of 7, 14, and 28 days postinjury (P<.001) among patients receiving cold-gel applications. Patients receiving cold-gel treatment also reported increased satisfaction with treatment compared with the controls. At 28 days, cold-gel treatment patients scored 71 on a 100-point satisfaction scale compared with 44 for controls (P<.001).3 Inconclusive results or significant design flaws limited the validity of all other trials cited in this review.3

Laboratory studies on rats have also demonstrated beneficial effects of cryotherapy after simulated soft-tissue injuries.4,5 One study cited a significant reduction in inflammatory cells, based on histologic examination, in 43 rats between 6 and 24 hours after trauma.4 A second study of 21 rats showed improvement in associated physiological components with cryotherapy, but no statistically significant improvement in edema.5

How cold is too cold?

Most authorities recommend empiric treatment with cryotherapy during the acute inflammatory phase—the first 24 to 48 hours after injury.6 Although not rigorously studied, some sources recommend applying cold to the involved muscle for the first 4 hours after injury at intervals of 10 to 20 minutes every 30 to 60 minutes.6

The literature focuses more on the optimal temperature for cryotherapy than on the duration and frequency of therapy.7 Temperatures below 15°to 25°C may actually result in vasodilatation rather than vasoconstriction.7

 

 

 

Evidence for heat is limited

A 2006 Cochrane review that addressed treatment of lower back muscular strain, not soft-tissue injuries in general, found moderate evidence that heat therapy reduces pain by 17% and disability in the acute setting (P=.001).8 The review also cited 2 head-to-head trials that compared heat and cryotherapy; however, the study designs were poor and the results were contradictory.8

Recommendations

Authoritative textbooks consistently recommend applying ice for initial treatment of musculoskeletal and soft-tissue strains.9

Acknowledgments

The opinions and assertions contained herein are the private views of the authors and not to be construed as official or as reflecting the views of the United States Air Force Medical Service or the United States Air Force at large.

References

1. Hubbard TJ, Denegar CR. Does cryotherapy improve outcomes with soft tissue injury? J Athl Train. 2004;39:278-279.

2. Kalenak A, Medlar CE, Fleagle SB, Hochberg WJ. Athletic injuries: heat vs cold. Am Fam Physician. 1975;12:131-134.

3. Collins NC. Is ice right? Does cryotherapy improve outcome for acute soft tissue injury? Emerg Med J. 2008;25:65-68.

4. Hurme T, Rantanen J, Kalimo H. Effects of early cryotherapy in experimental skeletal muscle injury. Scand J Med Sci Sports. 1993;3:46-51.

5. Schaser K, Disch AC, Stover JF, et al. Prolonged superficial local cryotherapy attenuates microcirculatory impairment, regional inflammation, and muscle necrosis after closed soft tissue injury in rats. Am J Sports Med. 2007;35:93-102.

6. Kellett J. Acute soft tissue injuries—a review of the literature. Med Sci Sports Exerc. 1986;18:489-500.

7. McMaster WC, Liddle S, Waugh TR. Laboratory evaluation of various cold therapy modalities. Am J Sports Med. 1978;6:291-294.

8. French SD, Cameron M, Walker BF, Reggars JW, Esterman AJ. A Cochrane review of superficial heat or cold for low back pain. Spine. 2006;31:998-1006.

9. Griffin LY. Essentials of Musculoskeletal Care. 3rd ed. Rosemont, Ill: American Academy of Orthopaedic Surgeons; 2005:134.

References

1. Hubbard TJ, Denegar CR. Does cryotherapy improve outcomes with soft tissue injury? J Athl Train. 2004;39:278-279.

2. Kalenak A, Medlar CE, Fleagle SB, Hochberg WJ. Athletic injuries: heat vs cold. Am Fam Physician. 1975;12:131-134.

3. Collins NC. Is ice right? Does cryotherapy improve outcome for acute soft tissue injury? Emerg Med J. 2008;25:65-68.

4. Hurme T, Rantanen J, Kalimo H. Effects of early cryotherapy in experimental skeletal muscle injury. Scand J Med Sci Sports. 1993;3:46-51.

5. Schaser K, Disch AC, Stover JF, et al. Prolonged superficial local cryotherapy attenuates microcirculatory impairment, regional inflammation, and muscle necrosis after closed soft tissue injury in rats. Am J Sports Med. 2007;35:93-102.

6. Kellett J. Acute soft tissue injuries—a review of the literature. Med Sci Sports Exerc. 1986;18:489-500.

7. McMaster WC, Liddle S, Waugh TR. Laboratory evaluation of various cold therapy modalities. Am J Sports Med. 1978;6:291-294.

8. French SD, Cameron M, Walker BF, Reggars JW, Esterman AJ. A Cochrane review of superficial heat or cold for low back pain. Spine. 2006;31:998-1006.

9. Griffin LY. Essentials of Musculoskeletal Care. 3rd ed. Rosemont, Ill: American Academy of Orthopaedic Surgeons; 2005:134.

Issue
The Journal of Family Practice - 57(12)
Issue
The Journal of Family Practice - 57(12)
Page Number
920-921
Page Number
920-921
Publications
Publications
Article Type
Display Headline
Does heat or cold work better for acute muscle strain?
Display Headline
Does heat or cold work better for acute muscle strain?
Legacy Keywords
Dean N. Martin; soft-tissue injury; cryotherapy; gel application; acute muscle strain
Legacy Keywords
Dean N. Martin; soft-tissue injury; cryotherapy; gel application; acute muscle strain
Sections
PURLs Copyright

Evidence-based answers from the Family Physicians Inquiries Network

Disallow All Ads
Article PDF Media

How much can exercise raise creatine kinase level—and does it matter?

Article Type
Changed
Mon, 01/14/2019 - 11:23
Display Headline
How much can exercise raise creatine kinase level—and does it matter?
EVIDENCE-BASED ANSWER

Moderate-intensity exercise (maintaining heart rate between 55% and 90% of maximum) may elevate creatine kinase (CK) to levels that meet the diagnostic criteria for rhabdomyolysis if the exercises involve eccentric muscle contractions, such as weight lifting or downhill running (strength of recommendation [SOR]: C, small observational studies). The clinical significance of exercise-induced elevations in CK is unclear because the renal complications associated with classic rhabdomyolysis haven’t been observed.

Clinical commentary

Be vigilant, but not hypervigilant
Tim Mott, MD
US Naval Hospital, Sigonella, Italy

Elevated CK noted on incidental testing can be vexing for physicians who treat athletes. Because asymptomatic exertional rhabdomyolysis is historically underdiagnosed and underappreciated, one may feel compelled to test all such patients for renal function, electrolytes, and myoglobinuria.1

Vigilance is mandatory—especially for symptoms of myalgia, generalized weakness, and dark urine—but this Clinical Inquiry also supports using a sound patient history and clinical judgment to avoid extensive laboratory testing or hospital admission. Indeed, patients who participate in moderate intensity, eccentric muscle contraction activities can be followed as outpatients because a correlation between CK elevation and renal dysfunction has not been detected in this group.

Evidence summary

Rhabdomyolysis is a well-described clinical syndrome resulting from injury to skeletal muscle and subsequent release of cellular contents into the extracellular fluid and circulation. It can lead to many complications, including renal failure, disseminated intravascular coagulation, and even death in 5% of cases.2 The leading causes of rhabdomyolysis include trauma, soft tissue compression, alcohol, drugs, infections, seizures, and exercise.3

Only half of patients experience muscle pain.2 Elevations occur in multiple serum markers, including CK, myoglobin, aldolase, lactate dehydrogenase, alanine aminotransferase, and aspartate aminotransferase, in either plasma or urine.4,5

Variable elevations, ranging from mild to extreme, that are discovered incidentally after exercise may cause clinical uncertainty.

 

No clear consensus defines CK levels in rhabdomyolysis

CK is the primary serum marker for rhabdomyolysis. It’s highly sensitive, but not specific. No clear consensus exists on what threshold of CK elevation correlates with clinically relevant disease.6 A relationship between CK elevation and the severity of disease has been established (>6000 IU/L predicts renal failure), but patients can have significant morbidity with only moderately elevated CK levels.7,8 Normal reference ranges for serum CK are 55 to 170 IU/L for males, and 30 to 135 IU/L for females.9

Recent definitions of rhabdomyolysis have been established to address muscle toxicity from lipid-lowering medications. The United States Food and Drug Administration specifies a CK level of more than 50 times the upper limit of normal (ULN)—or 10,000 IU/L—accompanied by organ damage, usually renal compromise.6 The National Lipid Association’s Muscle Safety Expert Panel has defined rhabdomyolysis as any evidence of muscle cell destruction regardless of the CK level and a causal relationship to a change in renal function. The panel further subdivides CK elevations into categories of mild (<10 times ULN), moderate (10-49 times ULN), and marked (≥50 times ULN).6

 

 

 

Exercise elevates CK level, but consider other factors, too

Although exercise is known to elevate CK, it produces a wide range of levels, based on a host of variables.3,10 Increases in CK are more pronounced in males, blacks, and untrained people; age doesn’t seem to be a factor.10,11 Higher-intensity, longer-duration, and weight-bearing exercise (eccentric muscular contractions and downhill running) cause the greatest rises in CK.10 Other influences include temperature, altitude, gravitational forces, noise, and vibration.

No studies firmly establish a normal range of CK elevation from moderate exercise; better data are available for extreme athletes, such as long-distance runners and triathletes. One study found that mean total CK elevations 24 hours after a marathon were 3322 IU/L (22.3 times baseline) for men and 946 IU/L (8.6 times baseline) for women.4 Another study showed that triathletes had a 12-fold mean increase in CK levels as long as 24 hours after the race.12

Eccentric exercises significantly raise CK

Exercise programs that include eccentric muscle contractions can result in significant serum CK elevations. One study followed 203 participants to evaluate the magnitude of CK elevation and the effect on renal function produced by exercise.3 After performing 50 maximal eccentric elbow flexor contractions, 55% of participants had CK elevations >2000 IU/L at 4 days after exercise; 25% had CK elevations >10,000 IU/L; 13% had levels >20,000 IU/L. None showed any evidence of renal compromise on clinical follow-up. Another study found significant increases in CK (approximate mean of 15,000 IU/L) after repetitive eccentric elbow flexor contractions in college-age males.13

Eccentric weight lifting and similar activities, like downhill running, may result in an increase in serum CK levels of 10 to 20 times normal, whereas other nonweight-bearing exercises and exercise involving no or minimal eccentric contractions, such as swimming and cycling, cause only nominal increases in serum CK.10

Recommendations

No formal guidelines from authoritative sources are available.

Acknowledgments

The opinions and assertions contained herein are the private views of the authors and not to be construed as official, or as reflecting the views of the US Air Force Medical Service or the US Air Force at large.

References

1. Chen TC, Hsieh SS. Effects of a 7-day eccentric training period on muscle damage and inflammation. Med Sci Sports Exerc. 2001;33:1732-1738.

2. Line R, Rust G. Acute exertional rhabdomyolysis. Am Fam Physician. 1995;52:502-506.

3. Rogers MA, Stull GA, Apple FS. Creatine kinase isoenzyme activities in men and women following a marathon race. Med Sci Sports Exerc. 1985;17:679-682.

4. Craig S. Rhabdomyolysis. November 2006. Available at: www.emedicine.com/emerg/topic508.htm. Accessed September 14, 2007.

5. Clarkson PM, Kearns AK, Rouzier P, Rubin R, Thompson PD. Serum creatine kinase levels and renal function measures in exertional muscle damage. Med Sci Sports Exerc. 2006;38:623-627.

6. Visweswaran P, Guntupalli J. Rhabdomyolysis. Crit Care Clin. 1999;15:415-428.

7. Thompson PD, Clarkson PM, Rosenson RS, et al. An assessment of statin safety by muscle experts. Am J Cardiol. 2006;97:69C-76C.

8. American College of Sports Medicine Position Stand. The recommended quantity and quality of exercise for developing and maintaining cardio-respiratory and muscular fitness and flexibility in healthy adults. Med Sci Sports Exerc. 1998;30:975-991.

9. Ward MM. Factors predictive of acute renal failure in rhabdomyolysis. Arch Intern Med. 1988;148:1553-1557.

10. McPherson RA, Pincus MR, eds. Henry’s Clinical Diagnosis and Management by Laboratory Methods. 21st ed. Philadelphia, Pa: Saunders Elsevier; 2007.

11. Noakes TD. Effect of exercise on serum enzyme activities in humans. Sports Med. 1987;4:245-267.

12. Munjal DD, McFadden JA, Matix PA, Coffman KD, Cattaneo SM. Changes in serum myoglobin, total creatine kinase, lactate dehydrogenase, and creatine kinase MB levels in runners. Clin Biochem. 1983;16:195-199.

13. Margaritis I, Tessier F, Verdera F, Bermon S, Marconnet P. Muscle enzyme release does not predict muscle function impairment after triathlon. J Sports Med Phys Fitness. 1999;39:133-139.

Article PDF
Author and Disclosure Information

Joshua Latham, DO;
Darren Campbell, MD
HQ Air Armament Center Family Medicine Residency, Eglin AFB, Fla

William Nichols, MLS
Eglin AFB, Fla

Issue
The Journal of Family Practice - 57(8)
Publications
Page Number
545-547
Legacy Keywords
creatine kinase; CK; rhabdomyolysis; triathlete; exercise-induced elevations in CK; Joshua Latham DO; Darren Campbell MD; William Nichols; Tim Mott MD
Sections
Author and Disclosure Information

Joshua Latham, DO;
Darren Campbell, MD
HQ Air Armament Center Family Medicine Residency, Eglin AFB, Fla

William Nichols, MLS
Eglin AFB, Fla

Author and Disclosure Information

Joshua Latham, DO;
Darren Campbell, MD
HQ Air Armament Center Family Medicine Residency, Eglin AFB, Fla

William Nichols, MLS
Eglin AFB, Fla

Article PDF
Article PDF
EVIDENCE-BASED ANSWER

Moderate-intensity exercise (maintaining heart rate between 55% and 90% of maximum) may elevate creatine kinase (CK) to levels that meet the diagnostic criteria for rhabdomyolysis if the exercises involve eccentric muscle contractions, such as weight lifting or downhill running (strength of recommendation [SOR]: C, small observational studies). The clinical significance of exercise-induced elevations in CK is unclear because the renal complications associated with classic rhabdomyolysis haven’t been observed.

Clinical commentary

Be vigilant, but not hypervigilant
Tim Mott, MD
US Naval Hospital, Sigonella, Italy

Elevated CK noted on incidental testing can be vexing for physicians who treat athletes. Because asymptomatic exertional rhabdomyolysis is historically underdiagnosed and underappreciated, one may feel compelled to test all such patients for renal function, electrolytes, and myoglobinuria.1

Vigilance is mandatory—especially for symptoms of myalgia, generalized weakness, and dark urine—but this Clinical Inquiry also supports using a sound patient history and clinical judgment to avoid extensive laboratory testing or hospital admission. Indeed, patients who participate in moderate intensity, eccentric muscle contraction activities can be followed as outpatients because a correlation between CK elevation and renal dysfunction has not been detected in this group.

Evidence summary

Rhabdomyolysis is a well-described clinical syndrome resulting from injury to skeletal muscle and subsequent release of cellular contents into the extracellular fluid and circulation. It can lead to many complications, including renal failure, disseminated intravascular coagulation, and even death in 5% of cases.2 The leading causes of rhabdomyolysis include trauma, soft tissue compression, alcohol, drugs, infections, seizures, and exercise.3

Only half of patients experience muscle pain.2 Elevations occur in multiple serum markers, including CK, myoglobin, aldolase, lactate dehydrogenase, alanine aminotransferase, and aspartate aminotransferase, in either plasma or urine.4,5

Variable elevations, ranging from mild to extreme, that are discovered incidentally after exercise may cause clinical uncertainty.

 

No clear consensus defines CK levels in rhabdomyolysis

CK is the primary serum marker for rhabdomyolysis. It’s highly sensitive, but not specific. No clear consensus exists on what threshold of CK elevation correlates with clinically relevant disease.6 A relationship between CK elevation and the severity of disease has been established (>6000 IU/L predicts renal failure), but patients can have significant morbidity with only moderately elevated CK levels.7,8 Normal reference ranges for serum CK are 55 to 170 IU/L for males, and 30 to 135 IU/L for females.9

Recent definitions of rhabdomyolysis have been established to address muscle toxicity from lipid-lowering medications. The United States Food and Drug Administration specifies a CK level of more than 50 times the upper limit of normal (ULN)—or 10,000 IU/L—accompanied by organ damage, usually renal compromise.6 The National Lipid Association’s Muscle Safety Expert Panel has defined rhabdomyolysis as any evidence of muscle cell destruction regardless of the CK level and a causal relationship to a change in renal function. The panel further subdivides CK elevations into categories of mild (<10 times ULN), moderate (10-49 times ULN), and marked (≥50 times ULN).6

 

 

 

Exercise elevates CK level, but consider other factors, too

Although exercise is known to elevate CK, it produces a wide range of levels, based on a host of variables.3,10 Increases in CK are more pronounced in males, blacks, and untrained people; age doesn’t seem to be a factor.10,11 Higher-intensity, longer-duration, and weight-bearing exercise (eccentric muscular contractions and downhill running) cause the greatest rises in CK.10 Other influences include temperature, altitude, gravitational forces, noise, and vibration.

No studies firmly establish a normal range of CK elevation from moderate exercise; better data are available for extreme athletes, such as long-distance runners and triathletes. One study found that mean total CK elevations 24 hours after a marathon were 3322 IU/L (22.3 times baseline) for men and 946 IU/L (8.6 times baseline) for women.4 Another study showed that triathletes had a 12-fold mean increase in CK levels as long as 24 hours after the race.12

Eccentric exercises significantly raise CK

Exercise programs that include eccentric muscle contractions can result in significant serum CK elevations. One study followed 203 participants to evaluate the magnitude of CK elevation and the effect on renal function produced by exercise.3 After performing 50 maximal eccentric elbow flexor contractions, 55% of participants had CK elevations >2000 IU/L at 4 days after exercise; 25% had CK elevations >10,000 IU/L; 13% had levels >20,000 IU/L. None showed any evidence of renal compromise on clinical follow-up. Another study found significant increases in CK (approximate mean of 15,000 IU/L) after repetitive eccentric elbow flexor contractions in college-age males.13

Eccentric weight lifting and similar activities, like downhill running, may result in an increase in serum CK levels of 10 to 20 times normal, whereas other nonweight-bearing exercises and exercise involving no or minimal eccentric contractions, such as swimming and cycling, cause only nominal increases in serum CK.10

Recommendations

No formal guidelines from authoritative sources are available.

Acknowledgments

The opinions and assertions contained herein are the private views of the authors and not to be construed as official, or as reflecting the views of the US Air Force Medical Service or the US Air Force at large.

EVIDENCE-BASED ANSWER

Moderate-intensity exercise (maintaining heart rate between 55% and 90% of maximum) may elevate creatine kinase (CK) to levels that meet the diagnostic criteria for rhabdomyolysis if the exercises involve eccentric muscle contractions, such as weight lifting or downhill running (strength of recommendation [SOR]: C, small observational studies). The clinical significance of exercise-induced elevations in CK is unclear because the renal complications associated with classic rhabdomyolysis haven’t been observed.

Clinical commentary

Be vigilant, but not hypervigilant
Tim Mott, MD
US Naval Hospital, Sigonella, Italy

Elevated CK noted on incidental testing can be vexing for physicians who treat athletes. Because asymptomatic exertional rhabdomyolysis is historically underdiagnosed and underappreciated, one may feel compelled to test all such patients for renal function, electrolytes, and myoglobinuria.1

Vigilance is mandatory—especially for symptoms of myalgia, generalized weakness, and dark urine—but this Clinical Inquiry also supports using a sound patient history and clinical judgment to avoid extensive laboratory testing or hospital admission. Indeed, patients who participate in moderate intensity, eccentric muscle contraction activities can be followed as outpatients because a correlation between CK elevation and renal dysfunction has not been detected in this group.

Evidence summary

Rhabdomyolysis is a well-described clinical syndrome resulting from injury to skeletal muscle and subsequent release of cellular contents into the extracellular fluid and circulation. It can lead to many complications, including renal failure, disseminated intravascular coagulation, and even death in 5% of cases.2 The leading causes of rhabdomyolysis include trauma, soft tissue compression, alcohol, drugs, infections, seizures, and exercise.3

Only half of patients experience muscle pain.2 Elevations occur in multiple serum markers, including CK, myoglobin, aldolase, lactate dehydrogenase, alanine aminotransferase, and aspartate aminotransferase, in either plasma or urine.4,5

Variable elevations, ranging from mild to extreme, that are discovered incidentally after exercise may cause clinical uncertainty.

 

No clear consensus defines CK levels in rhabdomyolysis

CK is the primary serum marker for rhabdomyolysis. It’s highly sensitive, but not specific. No clear consensus exists on what threshold of CK elevation correlates with clinically relevant disease.6 A relationship between CK elevation and the severity of disease has been established (>6000 IU/L predicts renal failure), but patients can have significant morbidity with only moderately elevated CK levels.7,8 Normal reference ranges for serum CK are 55 to 170 IU/L for males, and 30 to 135 IU/L for females.9

Recent definitions of rhabdomyolysis have been established to address muscle toxicity from lipid-lowering medications. The United States Food and Drug Administration specifies a CK level of more than 50 times the upper limit of normal (ULN)—or 10,000 IU/L—accompanied by organ damage, usually renal compromise.6 The National Lipid Association’s Muscle Safety Expert Panel has defined rhabdomyolysis as any evidence of muscle cell destruction regardless of the CK level and a causal relationship to a change in renal function. The panel further subdivides CK elevations into categories of mild (<10 times ULN), moderate (10-49 times ULN), and marked (≥50 times ULN).6

 

 

 

Exercise elevates CK level, but consider other factors, too

Although exercise is known to elevate CK, it produces a wide range of levels, based on a host of variables.3,10 Increases in CK are more pronounced in males, blacks, and untrained people; age doesn’t seem to be a factor.10,11 Higher-intensity, longer-duration, and weight-bearing exercise (eccentric muscular contractions and downhill running) cause the greatest rises in CK.10 Other influences include temperature, altitude, gravitational forces, noise, and vibration.

No studies firmly establish a normal range of CK elevation from moderate exercise; better data are available for extreme athletes, such as long-distance runners and triathletes. One study found that mean total CK elevations 24 hours after a marathon were 3322 IU/L (22.3 times baseline) for men and 946 IU/L (8.6 times baseline) for women.4 Another study showed that triathletes had a 12-fold mean increase in CK levels as long as 24 hours after the race.12

Eccentric exercises significantly raise CK

Exercise programs that include eccentric muscle contractions can result in significant serum CK elevations. One study followed 203 participants to evaluate the magnitude of CK elevation and the effect on renal function produced by exercise.3 After performing 50 maximal eccentric elbow flexor contractions, 55% of participants had CK elevations >2000 IU/L at 4 days after exercise; 25% had CK elevations >10,000 IU/L; 13% had levels >20,000 IU/L. None showed any evidence of renal compromise on clinical follow-up. Another study found significant increases in CK (approximate mean of 15,000 IU/L) after repetitive eccentric elbow flexor contractions in college-age males.13

Eccentric weight lifting and similar activities, like downhill running, may result in an increase in serum CK levels of 10 to 20 times normal, whereas other nonweight-bearing exercises and exercise involving no or minimal eccentric contractions, such as swimming and cycling, cause only nominal increases in serum CK.10

Recommendations

No formal guidelines from authoritative sources are available.

Acknowledgments

The opinions and assertions contained herein are the private views of the authors and not to be construed as official, or as reflecting the views of the US Air Force Medical Service or the US Air Force at large.

References

1. Chen TC, Hsieh SS. Effects of a 7-day eccentric training period on muscle damage and inflammation. Med Sci Sports Exerc. 2001;33:1732-1738.

2. Line R, Rust G. Acute exertional rhabdomyolysis. Am Fam Physician. 1995;52:502-506.

3. Rogers MA, Stull GA, Apple FS. Creatine kinase isoenzyme activities in men and women following a marathon race. Med Sci Sports Exerc. 1985;17:679-682.

4. Craig S. Rhabdomyolysis. November 2006. Available at: www.emedicine.com/emerg/topic508.htm. Accessed September 14, 2007.

5. Clarkson PM, Kearns AK, Rouzier P, Rubin R, Thompson PD. Serum creatine kinase levels and renal function measures in exertional muscle damage. Med Sci Sports Exerc. 2006;38:623-627.

6. Visweswaran P, Guntupalli J. Rhabdomyolysis. Crit Care Clin. 1999;15:415-428.

7. Thompson PD, Clarkson PM, Rosenson RS, et al. An assessment of statin safety by muscle experts. Am J Cardiol. 2006;97:69C-76C.

8. American College of Sports Medicine Position Stand. The recommended quantity and quality of exercise for developing and maintaining cardio-respiratory and muscular fitness and flexibility in healthy adults. Med Sci Sports Exerc. 1998;30:975-991.

9. Ward MM. Factors predictive of acute renal failure in rhabdomyolysis. Arch Intern Med. 1988;148:1553-1557.

10. McPherson RA, Pincus MR, eds. Henry’s Clinical Diagnosis and Management by Laboratory Methods. 21st ed. Philadelphia, Pa: Saunders Elsevier; 2007.

11. Noakes TD. Effect of exercise on serum enzyme activities in humans. Sports Med. 1987;4:245-267.

12. Munjal DD, McFadden JA, Matix PA, Coffman KD, Cattaneo SM. Changes in serum myoglobin, total creatine kinase, lactate dehydrogenase, and creatine kinase MB levels in runners. Clin Biochem. 1983;16:195-199.

13. Margaritis I, Tessier F, Verdera F, Bermon S, Marconnet P. Muscle enzyme release does not predict muscle function impairment after triathlon. J Sports Med Phys Fitness. 1999;39:133-139.

References

1. Chen TC, Hsieh SS. Effects of a 7-day eccentric training period on muscle damage and inflammation. Med Sci Sports Exerc. 2001;33:1732-1738.

2. Line R, Rust G. Acute exertional rhabdomyolysis. Am Fam Physician. 1995;52:502-506.

3. Rogers MA, Stull GA, Apple FS. Creatine kinase isoenzyme activities in men and women following a marathon race. Med Sci Sports Exerc. 1985;17:679-682.

4. Craig S. Rhabdomyolysis. November 2006. Available at: www.emedicine.com/emerg/topic508.htm. Accessed September 14, 2007.

5. Clarkson PM, Kearns AK, Rouzier P, Rubin R, Thompson PD. Serum creatine kinase levels and renal function measures in exertional muscle damage. Med Sci Sports Exerc. 2006;38:623-627.

6. Visweswaran P, Guntupalli J. Rhabdomyolysis. Crit Care Clin. 1999;15:415-428.

7. Thompson PD, Clarkson PM, Rosenson RS, et al. An assessment of statin safety by muscle experts. Am J Cardiol. 2006;97:69C-76C.

8. American College of Sports Medicine Position Stand. The recommended quantity and quality of exercise for developing and maintaining cardio-respiratory and muscular fitness and flexibility in healthy adults. Med Sci Sports Exerc. 1998;30:975-991.

9. Ward MM. Factors predictive of acute renal failure in rhabdomyolysis. Arch Intern Med. 1988;148:1553-1557.

10. McPherson RA, Pincus MR, eds. Henry’s Clinical Diagnosis and Management by Laboratory Methods. 21st ed. Philadelphia, Pa: Saunders Elsevier; 2007.

11. Noakes TD. Effect of exercise on serum enzyme activities in humans. Sports Med. 1987;4:245-267.

12. Munjal DD, McFadden JA, Matix PA, Coffman KD, Cattaneo SM. Changes in serum myoglobin, total creatine kinase, lactate dehydrogenase, and creatine kinase MB levels in runners. Clin Biochem. 1983;16:195-199.

13. Margaritis I, Tessier F, Verdera F, Bermon S, Marconnet P. Muscle enzyme release does not predict muscle function impairment after triathlon. J Sports Med Phys Fitness. 1999;39:133-139.

Issue
The Journal of Family Practice - 57(8)
Issue
The Journal of Family Practice - 57(8)
Page Number
545-547
Page Number
545-547
Publications
Publications
Article Type
Display Headline
How much can exercise raise creatine kinase level—and does it matter?
Display Headline
How much can exercise raise creatine kinase level—and does it matter?
Legacy Keywords
creatine kinase; CK; rhabdomyolysis; triathlete; exercise-induced elevations in CK; Joshua Latham DO; Darren Campbell MD; William Nichols; Tim Mott MD
Legacy Keywords
creatine kinase; CK; rhabdomyolysis; triathlete; exercise-induced elevations in CK; Joshua Latham DO; Darren Campbell MD; William Nichols; Tim Mott MD
Sections
PURLs Copyright

Evidence-based answers from the Family Physicians Inquiries Network

Disallow All Ads
Article PDF Media

What is the best way to screen for breast cancer in women with implants?

Article Type
Changed
Mon, 01/14/2019 - 11:23
Display Headline
What is the best way to screen for breast cancer in women with implants?
EVIDENCE-BASED ANSWER

Mammography is best. It is considered as effective for screening women who have undergone augmentation mammoplasty as those who have not (strength of recommendation [SOR]: B, limited number of retrospective and prospective cohort studies). This question has not been well studied, however.

Evidence summary

Breast augmentation is one of the most popular plastic surgeries in the United States; an estimated 291,350 such procedures were performed in 2005.1 Breast cancer occurs in 1 of every 8 women; a projected 32,000 women who received breast implants in 2003 will develop cancer.1 Available research has focused on retrospective and prospective designs because of the ethical limitations of experimental designs. No US studies directly compare mammography with alternate screening methods, such as sonography or magnetic resonance imaging.

With implants: Lower screening sensitivity but similar prognosis

Studies show that augmentation decreases the sensitivity of screening mammography but doesn’t affect breast cancer prognosis.2 A 2004 prospective cohort study of 986,270 women found that, among asymptomatic women diagnosed with breast cancer (40 augmented, 238 nonaugmented), the sensitivity of screening mammograms was lower in women with breast implants (45%; 95% confidence interval [CI], 29.3%–61.5%) than those without (66.8%; 95% CI, 60.4%–72.8%); P=.008).2 Similarly, in symptomatic women diagnosed with breast cancer (41 augmented, 145 nonaugmented), screening sensitivity was lower in the augmented women (73.2%) than the nonaugmented women (81.4%)—although the results weren’t significant (P=.25).

 

Despite lower screening sensitivity, breast tumors in asymptomatic women, whether augmented or not, had similar characteristics, except for larger tumor size (3 mm) at diagnosis in augmented women. Symptomatic women with breast implants had cancers that were smaller, lower-grade, and more likely to be estrogen receptor dependent and invasive (P=.052) compared with nonaugmented women. The authors concluded that augmentation doesn’t influence the prognostic characteristics of tumors, and they recommended screening mammography at appropriate intervals.

Two other prospective cohort studies produced similar findings. A 2006 study of 4082 breast cancer patients concluded that mammography yielded a false-negative rate of 41.4% in augmented patients compared with 8.8% in nonaugmented patients (P<.0001).3 However, both augmented (n=129) and nonaugmented (n=3953) women had a comparable prognosis at diagnosis. The authors of the studies suggested diagnostic mammography for augmented patients and correlation with physical exam findings.

An earlier study of 2956 cancer patients found that mammography detected an abnormal breast mass in 66.3% of augmented women compared with 94.6% of nonaugmented women (P=.001).4 No significant differences were noted in cancer characteristics at diagnosis or survival rates (P=.78). The authors of this study concluded that mammography should be used for augmented women until a more effective screening tool is found.

 

 

 

Sonography vs mammography: The jury is still out

Although studies comparing screening methods have not been performed in the United States, a small Taiwanese study directly compared ultrasound to mammography in 105 women without breast implants. This retrospective cohort study found sonography to be a more useful diagnostic tool than mammography in Taiwanese women.5 Sonography had the highest sensitivity (87.5%) compared to physical examination (50.0%) and mammography (25.5%).

Sonography was recommended as the imaging tool for Asian women with smaller, denser breasts. However, it is unclear whether this result applies to US women or women who have undergone breast augmentation surgery.

Training in implant imaging is needed

Mammography appears to be the most effective screening method for women with breast implants. Despite the small differences in cancer characteristics at diagnosis between augmented and nonaugmented women, overall prognosis and survival rates are no different.1-3,6 This is true whether a screening mammogram or diagnostic mammogram is used. In any case, all available findings suggest that clinicians who perform mammography should be trained in imaging the augmented breast.6-8

Recommendations

The National Cancer Institute indicates that the best screening method for augmented women is mammography performed at a facility with employees trained in implant imaging.7 The American College of Radiology’s practice guidelines affirm that mammography is the best imaging tool available.8 The American College of Obstetrics and Gynecology and the US Preventive Services Task Force don’t comment on screening augmented women.

Acknowledgments

The opinions and assertions contained herein are the private views of the author and not to be construed as official, or as reflecting the views of the US Air Force Medical Service or the US Air Force at large.

References

1. Tuli R, Flynn RA, Brill KL, et al. Diagnosis, treatment, and management of breast cancer in previously augmented women. Breast J. 2006;12:343-348.

2. Miglioretti DL, Rutter CM, Gellar BM, et al. Effect of breast augmentation on the accuracy of mammography and cancer characteristics. JAMA. 2004;291:442-450.

3. Handel N, Silverstein MJ. Breast cancer diagnosis and prognosis in augmented women. Plast Reconstr Surg. 2006;118:587-593.

4. Skinner KA, Silberman H, Dougherty W, et al. Breast cancer after augmentation mammoplasty. Ann Surg Oncol. 2000;8:138-144.

5. Hou M-F, Ou-Yang F, Chuang C-H, et al. Comparison between sonography and mammography for breast cancer diagnosis in oriental women after augmentation mammaplasty. Ann Plast Surg. 2002;49:120-126.

6. Hoshaw SJ, Klein PJ, Clark BD, et al. Breast implants and cancer: causation, delayed detection, and survival. Plast Reconstr Surg. 2001;107:1393-1407.

7. National Cancer Institute. Screening Mammograms: Questions and Answers. September 4, 2007. Available at: www.cancer.gov/cancertopics/factsheet/Detection/screening-mammograms. Accessed November 2, 2007.

8. American College of Radiology. ACR Practice Guideline (amended 2006). Available at: www.acr.org/SecondaryMainMenuCategories/quality_safety/guidelines/breast/screening_mammography.aspx. Accessed November 2, 2007.

Article PDF
Author and Disclosure Information

Patricia Goodemote, MD
Deana Mitchell, DO
Eglin Air Force Base Family Medicine Residency, Eglin AFB, Fla

William Nichols, MLS
Eglin Air Force Base, Fla

Issue
The Journal of Family Practice - 57(7)
Publications
Topics
Page Number
482-483
Legacy Keywords
breast cancer; breast augmentation; mammography
Sections
Author and Disclosure Information

Patricia Goodemote, MD
Deana Mitchell, DO
Eglin Air Force Base Family Medicine Residency, Eglin AFB, Fla

William Nichols, MLS
Eglin Air Force Base, Fla

Author and Disclosure Information

Patricia Goodemote, MD
Deana Mitchell, DO
Eglin Air Force Base Family Medicine Residency, Eglin AFB, Fla

William Nichols, MLS
Eglin Air Force Base, Fla

Article PDF
Article PDF
EVIDENCE-BASED ANSWER

Mammography is best. It is considered as effective for screening women who have undergone augmentation mammoplasty as those who have not (strength of recommendation [SOR]: B, limited number of retrospective and prospective cohort studies). This question has not been well studied, however.

Evidence summary

Breast augmentation is one of the most popular plastic surgeries in the United States; an estimated 291,350 such procedures were performed in 2005.1 Breast cancer occurs in 1 of every 8 women; a projected 32,000 women who received breast implants in 2003 will develop cancer.1 Available research has focused on retrospective and prospective designs because of the ethical limitations of experimental designs. No US studies directly compare mammography with alternate screening methods, such as sonography or magnetic resonance imaging.

With implants: Lower screening sensitivity but similar prognosis

Studies show that augmentation decreases the sensitivity of screening mammography but doesn’t affect breast cancer prognosis.2 A 2004 prospective cohort study of 986,270 women found that, among asymptomatic women diagnosed with breast cancer (40 augmented, 238 nonaugmented), the sensitivity of screening mammograms was lower in women with breast implants (45%; 95% confidence interval [CI], 29.3%–61.5%) than those without (66.8%; 95% CI, 60.4%–72.8%); P=.008).2 Similarly, in symptomatic women diagnosed with breast cancer (41 augmented, 145 nonaugmented), screening sensitivity was lower in the augmented women (73.2%) than the nonaugmented women (81.4%)—although the results weren’t significant (P=.25).

 

Despite lower screening sensitivity, breast tumors in asymptomatic women, whether augmented or not, had similar characteristics, except for larger tumor size (3 mm) at diagnosis in augmented women. Symptomatic women with breast implants had cancers that were smaller, lower-grade, and more likely to be estrogen receptor dependent and invasive (P=.052) compared with nonaugmented women. The authors concluded that augmentation doesn’t influence the prognostic characteristics of tumors, and they recommended screening mammography at appropriate intervals.

Two other prospective cohort studies produced similar findings. A 2006 study of 4082 breast cancer patients concluded that mammography yielded a false-negative rate of 41.4% in augmented patients compared with 8.8% in nonaugmented patients (P<.0001).3 However, both augmented (n=129) and nonaugmented (n=3953) women had a comparable prognosis at diagnosis. The authors of the studies suggested diagnostic mammography for augmented patients and correlation with physical exam findings.

An earlier study of 2956 cancer patients found that mammography detected an abnormal breast mass in 66.3% of augmented women compared with 94.6% of nonaugmented women (P=.001).4 No significant differences were noted in cancer characteristics at diagnosis or survival rates (P=.78). The authors of this study concluded that mammography should be used for augmented women until a more effective screening tool is found.

 

 

 

Sonography vs mammography: The jury is still out

Although studies comparing screening methods have not been performed in the United States, a small Taiwanese study directly compared ultrasound to mammography in 105 women without breast implants. This retrospective cohort study found sonography to be a more useful diagnostic tool than mammography in Taiwanese women.5 Sonography had the highest sensitivity (87.5%) compared to physical examination (50.0%) and mammography (25.5%).

Sonography was recommended as the imaging tool for Asian women with smaller, denser breasts. However, it is unclear whether this result applies to US women or women who have undergone breast augmentation surgery.

Training in implant imaging is needed

Mammography appears to be the most effective screening method for women with breast implants. Despite the small differences in cancer characteristics at diagnosis between augmented and nonaugmented women, overall prognosis and survival rates are no different.1-3,6 This is true whether a screening mammogram or diagnostic mammogram is used. In any case, all available findings suggest that clinicians who perform mammography should be trained in imaging the augmented breast.6-8

Recommendations

The National Cancer Institute indicates that the best screening method for augmented women is mammography performed at a facility with employees trained in implant imaging.7 The American College of Radiology’s practice guidelines affirm that mammography is the best imaging tool available.8 The American College of Obstetrics and Gynecology and the US Preventive Services Task Force don’t comment on screening augmented women.

Acknowledgments

The opinions and assertions contained herein are the private views of the author and not to be construed as official, or as reflecting the views of the US Air Force Medical Service or the US Air Force at large.

EVIDENCE-BASED ANSWER

Mammography is best. It is considered as effective for screening women who have undergone augmentation mammoplasty as those who have not (strength of recommendation [SOR]: B, limited number of retrospective and prospective cohort studies). This question has not been well studied, however.

Evidence summary

Breast augmentation is one of the most popular plastic surgeries in the United States; an estimated 291,350 such procedures were performed in 2005.1 Breast cancer occurs in 1 of every 8 women; a projected 32,000 women who received breast implants in 2003 will develop cancer.1 Available research has focused on retrospective and prospective designs because of the ethical limitations of experimental designs. No US studies directly compare mammography with alternate screening methods, such as sonography or magnetic resonance imaging.

With implants: Lower screening sensitivity but similar prognosis

Studies show that augmentation decreases the sensitivity of screening mammography but doesn’t affect breast cancer prognosis.2 A 2004 prospective cohort study of 986,270 women found that, among asymptomatic women diagnosed with breast cancer (40 augmented, 238 nonaugmented), the sensitivity of screening mammograms was lower in women with breast implants (45%; 95% confidence interval [CI], 29.3%–61.5%) than those without (66.8%; 95% CI, 60.4%–72.8%); P=.008).2 Similarly, in symptomatic women diagnosed with breast cancer (41 augmented, 145 nonaugmented), screening sensitivity was lower in the augmented women (73.2%) than the nonaugmented women (81.4%)—although the results weren’t significant (P=.25).

 

Despite lower screening sensitivity, breast tumors in asymptomatic women, whether augmented or not, had similar characteristics, except for larger tumor size (3 mm) at diagnosis in augmented women. Symptomatic women with breast implants had cancers that were smaller, lower-grade, and more likely to be estrogen receptor dependent and invasive (P=.052) compared with nonaugmented women. The authors concluded that augmentation doesn’t influence the prognostic characteristics of tumors, and they recommended screening mammography at appropriate intervals.

Two other prospective cohort studies produced similar findings. A 2006 study of 4082 breast cancer patients concluded that mammography yielded a false-negative rate of 41.4% in augmented patients compared with 8.8% in nonaugmented patients (P<.0001).3 However, both augmented (n=129) and nonaugmented (n=3953) women had a comparable prognosis at diagnosis. The authors of the studies suggested diagnostic mammography for augmented patients and correlation with physical exam findings.

An earlier study of 2956 cancer patients found that mammography detected an abnormal breast mass in 66.3% of augmented women compared with 94.6% of nonaugmented women (P=.001).4 No significant differences were noted in cancer characteristics at diagnosis or survival rates (P=.78). The authors of this study concluded that mammography should be used for augmented women until a more effective screening tool is found.

 

 

 

Sonography vs mammography: The jury is still out

Although studies comparing screening methods have not been performed in the United States, a small Taiwanese study directly compared ultrasound to mammography in 105 women without breast implants. This retrospective cohort study found sonography to be a more useful diagnostic tool than mammography in Taiwanese women.5 Sonography had the highest sensitivity (87.5%) compared to physical examination (50.0%) and mammography (25.5%).

Sonography was recommended as the imaging tool for Asian women with smaller, denser breasts. However, it is unclear whether this result applies to US women or women who have undergone breast augmentation surgery.

Training in implant imaging is needed

Mammography appears to be the most effective screening method for women with breast implants. Despite the small differences in cancer characteristics at diagnosis between augmented and nonaugmented women, overall prognosis and survival rates are no different.1-3,6 This is true whether a screening mammogram or diagnostic mammogram is used. In any case, all available findings suggest that clinicians who perform mammography should be trained in imaging the augmented breast.6-8

Recommendations

The National Cancer Institute indicates that the best screening method for augmented women is mammography performed at a facility with employees trained in implant imaging.7 The American College of Radiology’s practice guidelines affirm that mammography is the best imaging tool available.8 The American College of Obstetrics and Gynecology and the US Preventive Services Task Force don’t comment on screening augmented women.

Acknowledgments

The opinions and assertions contained herein are the private views of the author and not to be construed as official, or as reflecting the views of the US Air Force Medical Service or the US Air Force at large.

References

1. Tuli R, Flynn RA, Brill KL, et al. Diagnosis, treatment, and management of breast cancer in previously augmented women. Breast J. 2006;12:343-348.

2. Miglioretti DL, Rutter CM, Gellar BM, et al. Effect of breast augmentation on the accuracy of mammography and cancer characteristics. JAMA. 2004;291:442-450.

3. Handel N, Silverstein MJ. Breast cancer diagnosis and prognosis in augmented women. Plast Reconstr Surg. 2006;118:587-593.

4. Skinner KA, Silberman H, Dougherty W, et al. Breast cancer after augmentation mammoplasty. Ann Surg Oncol. 2000;8:138-144.

5. Hou M-F, Ou-Yang F, Chuang C-H, et al. Comparison between sonography and mammography for breast cancer diagnosis in oriental women after augmentation mammaplasty. Ann Plast Surg. 2002;49:120-126.

6. Hoshaw SJ, Klein PJ, Clark BD, et al. Breast implants and cancer: causation, delayed detection, and survival. Plast Reconstr Surg. 2001;107:1393-1407.

7. National Cancer Institute. Screening Mammograms: Questions and Answers. September 4, 2007. Available at: www.cancer.gov/cancertopics/factsheet/Detection/screening-mammograms. Accessed November 2, 2007.

8. American College of Radiology. ACR Practice Guideline (amended 2006). Available at: www.acr.org/SecondaryMainMenuCategories/quality_safety/guidelines/breast/screening_mammography.aspx. Accessed November 2, 2007.

References

1. Tuli R, Flynn RA, Brill KL, et al. Diagnosis, treatment, and management of breast cancer in previously augmented women. Breast J. 2006;12:343-348.

2. Miglioretti DL, Rutter CM, Gellar BM, et al. Effect of breast augmentation on the accuracy of mammography and cancer characteristics. JAMA. 2004;291:442-450.

3. Handel N, Silverstein MJ. Breast cancer diagnosis and prognosis in augmented women. Plast Reconstr Surg. 2006;118:587-593.

4. Skinner KA, Silberman H, Dougherty W, et al. Breast cancer after augmentation mammoplasty. Ann Surg Oncol. 2000;8:138-144.

5. Hou M-F, Ou-Yang F, Chuang C-H, et al. Comparison between sonography and mammography for breast cancer diagnosis in oriental women after augmentation mammaplasty. Ann Plast Surg. 2002;49:120-126.

6. Hoshaw SJ, Klein PJ, Clark BD, et al. Breast implants and cancer: causation, delayed detection, and survival. Plast Reconstr Surg. 2001;107:1393-1407.

7. National Cancer Institute. Screening Mammograms: Questions and Answers. September 4, 2007. Available at: www.cancer.gov/cancertopics/factsheet/Detection/screening-mammograms. Accessed November 2, 2007.

8. American College of Radiology. ACR Practice Guideline (amended 2006). Available at: www.acr.org/SecondaryMainMenuCategories/quality_safety/guidelines/breast/screening_mammography.aspx. Accessed November 2, 2007.

Issue
The Journal of Family Practice - 57(7)
Issue
The Journal of Family Practice - 57(7)
Page Number
482-483
Page Number
482-483
Publications
Publications
Topics
Article Type
Display Headline
What is the best way to screen for breast cancer in women with implants?
Display Headline
What is the best way to screen for breast cancer in women with implants?
Legacy Keywords
breast cancer; breast augmentation; mammography
Legacy Keywords
breast cancer; breast augmentation; mammography
Sections
PURLs Copyright

Evidence-based answers from the Family Physicians Inquiries Network

Disallow All Ads
Alternative CME
Article PDF Media

Does birth weight predict childhood obesity?

Article Type
Changed
Mon, 01/14/2019 - 11:23
Display Headline
Does birth weight predict childhood obesity?
EVIDENCE-BASED ANSWER

Yes. A birth weight greater than 4,000 g is associated with an increased risk of obesity in both childhood and adolescence (strength of recommendation [SOR]: B, systematic review and multiple cohort studies).

Clinical commentary

Lifestyle matters, too
David Krulak, MD, MPH
Camp Lejeune, NC

Few people have more questions than brand-new parents. Physicians often answer these inquiries from their pool of clinical experience or pearls handed down by mentors. It’s refreshing to be able to address a parental query on the basis of good evidence rather than empiricism.

The data compel us to inform parents that a new baby who weighs more than 4 kg is at increased risk of childhood obesity. However, all parents should be counseled that the lifestyle choices they make for their child are far more likely than birth weight to influence future obesity. Education about appropriate diet and physical activity is the bedrock from which to attack the childhood obesity epidemic.

Evidence summary

The number of children 2 years and older who are overweight has tripled in the past 2 decades; the current prevalence of over-weight children and adolescents in the US is 15%.1 By contrast with adults—in whom overweight and obesity are defined separately as a body mass index (BMI) above 25 kg/m2 and 30 kg/m2, respectively—overweight and obesity are synonymous in children and are defined as a BMI above the 95th percentile for age and sex.2 Children and adolescents with a BMI between the 85th and 95th percentiles are considered at risk for overweight.2

Overweight children are vulnerable to adverse health outcomes, including insulin resistance, hyperlipidemia, hypertension, depression, sleep apnea, asthma, steatohepatitis, genu varum, and slipped capital femoral epiphysis.2 The many variables that have been suggested to influence childhood obesity include birth weight, gestational age, parental obesity, socioeconomic status, single parent household, and birth order.3-5

 

Birth weight and later BMI: Consistently linked

A systematic review of 19 longitudinal, observational studies comparing birth weight with later BMI indicates that the association between the 2 is positive and consistent in multiple cohorts.3 Eleven studies focused on outcomes in childhood; another 8 measured outcomes into adulthood.

Fifteen of the 19 studies (79%), ranging in size from 1028 to 92,940 subjects, found a positive association between birth weight and later BMI. However, the data were too heterogeneous to combine into a single summary measure. One representative study quantified the relative risk (RR) for severe obesity (>95th percentile BMI) at 5 years of age as 1.7 (95% confidence interval [CI], 1.2-2.9) for birth weights between the 85th and 94th percentiles and 1.8 (95% CI, 1.1-2.9) for birth weights greater than the 95th percentile.3 Studies that didn’t find such an association had smaller sample sizes (137 to 432 subjects) and, therefore, may have lacked the power to detect an association.

 

 

Gestational diabetes. A subsequent retrospective cohort survey of 14,881 children born to mothers with gestational diabetes—and controlled for age, sex, and Tanner stage—found that the odds ratio (OR) for adolescent overweight was 1.4 (95% CI, 1.2-1.6) for each 1-kg increment in birth weight.4 The correlation persisted (OR=1.3; 95% CI, 1.1-1.5) when other covariates were controlled (television viewing, physical activity, energy intake, breastfeeding duration, birth order, household income, mother’s smoking, dietary restraint, and mother’s current BMI).

Large for gestational age. A US national cohort study of 3192 children adjusted for gestational age, found that large-for-gestational-age (LGA) infants with birth weights above the 90th percentile remained longer and heavier through 83 months of life.5 The triceps and subscapular skinfold measurements at 3 years of age for children born LGA were virtually identical to those of children born appropriate for gestational age, but by 6 years of age, skinfold measurements had diverged considerably, to more than 0.60 standard deviations. The researchers concluded that intrauterine growth is associated with obesity in early childhood.

Finally, a large Chinese population-based, case-control study (N=1322), found birth weight above 4.0 kg to be a risk factor for obesity in preschool-age children (OR=3.77; 95% CI, 2.06-6.29).6 The absolute rate of overweight increased from 8% to 26% among LGA infants.

 

In adolescence, parental weight may be a factor

A prospective cohort study of 1993 white LGA infants found a greater propensity to become obese in adolescence, but only if their mothers or fathers were also obese (RR=5.7).7 Children with lean parents did not have an increased risk of being over-weight in adolescence.

Recommendations

Although major organizations don’t focus on infant birth weight as a predictor of overweight, they do address childhood obesity. The American Academy of Pediatrics states that genetic, environmental, or combinations of risk factors predisposing children to obesity can and should be identified.2 The US Preventive Services Task Force concludes that the evidence is insufficient to recommend for or against routine screening for overweight in children and adolescents as a means of preventing adverse health outcomes (Grade I recommendation).1

Acknowledgements

The opinions and assertions contained herein are the private view of the author and not to be construed as official or as reflecting the view of the US Air Force Medical Service or the US Air Force at large.

References

1. US Preventive Services Task Force Screening and interventions for childhood obesity. Guide to Clinical Preventive Services. Rockville, MD: Agency for Healthcare Research and Quality; 2005.

2. Krebs NF, Jacobson MS. and the American Academy of Pediatrics Committee on Nutrition. Prevention of pediatric overweight and obesity. Pediatrics. 2003;112:424-430.

3. Parsons TJ, Power C, Logan S, et al. Childhood predictors of adult obesity: a systematic review. Int J Obes Relat Metab Disord. 1999;23(suppl 8):S1-S107.

4. Gillman MW, Rifas-Shiman S, Berkey CS, et al. Maternal gestational diabetes, birth weight, and adolescent obesity. Pediatrics. 2003;111(3):e221-226.

5. Hediger ML, Overpeck MD, McGlynn A, et al. Growth and fatness at three to six years of age of children born small- or large-for-gestational age. Pediatrics. 1999;104:e33.-

6. He Q, Ding ZY, Fong DY, et al. Risk factors of obesity in preschool children in China: a population-based case-control study. Int J Obes Relat Metab Disord. 2000;24:1528-1536.

7. Frisancho AR. Prenatal compared with parental origins of adolescent fatness. Am J Clin Nutr. 2000;72:1186-1190.

Article PDF
Author and Disclosure Information

Robert K Persons, DO
Tristan L. Sevdy, MD, FAAFP
Eglin Family Medicine Residency, Eglin Air Force Base, Fla

William Nichols, MLS
Eglin Air Force Base, Fla

Issue
The Journal of Family Practice - 57(6)
Publications
Topics
Page Number
409-410
Legacy Keywords
birth weight; childhood obesity; overweight; Robert K. Persons DO; Tristan L. Sevdy MD; David Krulak MD MPH
Sections
Author and Disclosure Information

Robert K Persons, DO
Tristan L. Sevdy, MD, FAAFP
Eglin Family Medicine Residency, Eglin Air Force Base, Fla

William Nichols, MLS
Eglin Air Force Base, Fla

Author and Disclosure Information

Robert K Persons, DO
Tristan L. Sevdy, MD, FAAFP
Eglin Family Medicine Residency, Eglin Air Force Base, Fla

William Nichols, MLS
Eglin Air Force Base, Fla

Article PDF
Article PDF
EVIDENCE-BASED ANSWER

Yes. A birth weight greater than 4,000 g is associated with an increased risk of obesity in both childhood and adolescence (strength of recommendation [SOR]: B, systematic review and multiple cohort studies).

Clinical commentary

Lifestyle matters, too
David Krulak, MD, MPH
Camp Lejeune, NC

Few people have more questions than brand-new parents. Physicians often answer these inquiries from their pool of clinical experience or pearls handed down by mentors. It’s refreshing to be able to address a parental query on the basis of good evidence rather than empiricism.

The data compel us to inform parents that a new baby who weighs more than 4 kg is at increased risk of childhood obesity. However, all parents should be counseled that the lifestyle choices they make for their child are far more likely than birth weight to influence future obesity. Education about appropriate diet and physical activity is the bedrock from which to attack the childhood obesity epidemic.

Evidence summary

The number of children 2 years and older who are overweight has tripled in the past 2 decades; the current prevalence of over-weight children and adolescents in the US is 15%.1 By contrast with adults—in whom overweight and obesity are defined separately as a body mass index (BMI) above 25 kg/m2 and 30 kg/m2, respectively—overweight and obesity are synonymous in children and are defined as a BMI above the 95th percentile for age and sex.2 Children and adolescents with a BMI between the 85th and 95th percentiles are considered at risk for overweight.2

Overweight children are vulnerable to adverse health outcomes, including insulin resistance, hyperlipidemia, hypertension, depression, sleep apnea, asthma, steatohepatitis, genu varum, and slipped capital femoral epiphysis.2 The many variables that have been suggested to influence childhood obesity include birth weight, gestational age, parental obesity, socioeconomic status, single parent household, and birth order.3-5

 

Birth weight and later BMI: Consistently linked

A systematic review of 19 longitudinal, observational studies comparing birth weight with later BMI indicates that the association between the 2 is positive and consistent in multiple cohorts.3 Eleven studies focused on outcomes in childhood; another 8 measured outcomes into adulthood.

Fifteen of the 19 studies (79%), ranging in size from 1028 to 92,940 subjects, found a positive association between birth weight and later BMI. However, the data were too heterogeneous to combine into a single summary measure. One representative study quantified the relative risk (RR) for severe obesity (>95th percentile BMI) at 5 years of age as 1.7 (95% confidence interval [CI], 1.2-2.9) for birth weights between the 85th and 94th percentiles and 1.8 (95% CI, 1.1-2.9) for birth weights greater than the 95th percentile.3 Studies that didn’t find such an association had smaller sample sizes (137 to 432 subjects) and, therefore, may have lacked the power to detect an association.

 

 

Gestational diabetes. A subsequent retrospective cohort survey of 14,881 children born to mothers with gestational diabetes—and controlled for age, sex, and Tanner stage—found that the odds ratio (OR) for adolescent overweight was 1.4 (95% CI, 1.2-1.6) for each 1-kg increment in birth weight.4 The correlation persisted (OR=1.3; 95% CI, 1.1-1.5) when other covariates were controlled (television viewing, physical activity, energy intake, breastfeeding duration, birth order, household income, mother’s smoking, dietary restraint, and mother’s current BMI).

Large for gestational age. A US national cohort study of 3192 children adjusted for gestational age, found that large-for-gestational-age (LGA) infants with birth weights above the 90th percentile remained longer and heavier through 83 months of life.5 The triceps and subscapular skinfold measurements at 3 years of age for children born LGA were virtually identical to those of children born appropriate for gestational age, but by 6 years of age, skinfold measurements had diverged considerably, to more than 0.60 standard deviations. The researchers concluded that intrauterine growth is associated with obesity in early childhood.

Finally, a large Chinese population-based, case-control study (N=1322), found birth weight above 4.0 kg to be a risk factor for obesity in preschool-age children (OR=3.77; 95% CI, 2.06-6.29).6 The absolute rate of overweight increased from 8% to 26% among LGA infants.

 

In adolescence, parental weight may be a factor

A prospective cohort study of 1993 white LGA infants found a greater propensity to become obese in adolescence, but only if their mothers or fathers were also obese (RR=5.7).7 Children with lean parents did not have an increased risk of being over-weight in adolescence.

Recommendations

Although major organizations don’t focus on infant birth weight as a predictor of overweight, they do address childhood obesity. The American Academy of Pediatrics states that genetic, environmental, or combinations of risk factors predisposing children to obesity can and should be identified.2 The US Preventive Services Task Force concludes that the evidence is insufficient to recommend for or against routine screening for overweight in children and adolescents as a means of preventing adverse health outcomes (Grade I recommendation).1

Acknowledgements

The opinions and assertions contained herein are the private view of the author and not to be construed as official or as reflecting the view of the US Air Force Medical Service or the US Air Force at large.

EVIDENCE-BASED ANSWER

Yes. A birth weight greater than 4,000 g is associated with an increased risk of obesity in both childhood and adolescence (strength of recommendation [SOR]: B, systematic review and multiple cohort studies).

Clinical commentary

Lifestyle matters, too
David Krulak, MD, MPH
Camp Lejeune, NC

Few people have more questions than brand-new parents. Physicians often answer these inquiries from their pool of clinical experience or pearls handed down by mentors. It’s refreshing to be able to address a parental query on the basis of good evidence rather than empiricism.

The data compel us to inform parents that a new baby who weighs more than 4 kg is at increased risk of childhood obesity. However, all parents should be counseled that the lifestyle choices they make for their child are far more likely than birth weight to influence future obesity. Education about appropriate diet and physical activity is the bedrock from which to attack the childhood obesity epidemic.

Evidence summary

The number of children 2 years and older who are overweight has tripled in the past 2 decades; the current prevalence of over-weight children and adolescents in the US is 15%.1 By contrast with adults—in whom overweight and obesity are defined separately as a body mass index (BMI) above 25 kg/m2 and 30 kg/m2, respectively—overweight and obesity are synonymous in children and are defined as a BMI above the 95th percentile for age and sex.2 Children and adolescents with a BMI between the 85th and 95th percentiles are considered at risk for overweight.2

Overweight children are vulnerable to adverse health outcomes, including insulin resistance, hyperlipidemia, hypertension, depression, sleep apnea, asthma, steatohepatitis, genu varum, and slipped capital femoral epiphysis.2 The many variables that have been suggested to influence childhood obesity include birth weight, gestational age, parental obesity, socioeconomic status, single parent household, and birth order.3-5

 

Birth weight and later BMI: Consistently linked

A systematic review of 19 longitudinal, observational studies comparing birth weight with later BMI indicates that the association between the 2 is positive and consistent in multiple cohorts.3 Eleven studies focused on outcomes in childhood; another 8 measured outcomes into adulthood.

Fifteen of the 19 studies (79%), ranging in size from 1028 to 92,940 subjects, found a positive association between birth weight and later BMI. However, the data were too heterogeneous to combine into a single summary measure. One representative study quantified the relative risk (RR) for severe obesity (>95th percentile BMI) at 5 years of age as 1.7 (95% confidence interval [CI], 1.2-2.9) for birth weights between the 85th and 94th percentiles and 1.8 (95% CI, 1.1-2.9) for birth weights greater than the 95th percentile.3 Studies that didn’t find such an association had smaller sample sizes (137 to 432 subjects) and, therefore, may have lacked the power to detect an association.

 

 

Gestational diabetes. A subsequent retrospective cohort survey of 14,881 children born to mothers with gestational diabetes—and controlled for age, sex, and Tanner stage—found that the odds ratio (OR) for adolescent overweight was 1.4 (95% CI, 1.2-1.6) for each 1-kg increment in birth weight.4 The correlation persisted (OR=1.3; 95% CI, 1.1-1.5) when other covariates were controlled (television viewing, physical activity, energy intake, breastfeeding duration, birth order, household income, mother’s smoking, dietary restraint, and mother’s current BMI).

Large for gestational age. A US national cohort study of 3192 children adjusted for gestational age, found that large-for-gestational-age (LGA) infants with birth weights above the 90th percentile remained longer and heavier through 83 months of life.5 The triceps and subscapular skinfold measurements at 3 years of age for children born LGA were virtually identical to those of children born appropriate for gestational age, but by 6 years of age, skinfold measurements had diverged considerably, to more than 0.60 standard deviations. The researchers concluded that intrauterine growth is associated with obesity in early childhood.

Finally, a large Chinese population-based, case-control study (N=1322), found birth weight above 4.0 kg to be a risk factor for obesity in preschool-age children (OR=3.77; 95% CI, 2.06-6.29).6 The absolute rate of overweight increased from 8% to 26% among LGA infants.

 

In adolescence, parental weight may be a factor

A prospective cohort study of 1993 white LGA infants found a greater propensity to become obese in adolescence, but only if their mothers or fathers were also obese (RR=5.7).7 Children with lean parents did not have an increased risk of being over-weight in adolescence.

Recommendations

Although major organizations don’t focus on infant birth weight as a predictor of overweight, they do address childhood obesity. The American Academy of Pediatrics states that genetic, environmental, or combinations of risk factors predisposing children to obesity can and should be identified.2 The US Preventive Services Task Force concludes that the evidence is insufficient to recommend for or against routine screening for overweight in children and adolescents as a means of preventing adverse health outcomes (Grade I recommendation).1

Acknowledgements

The opinions and assertions contained herein are the private view of the author and not to be construed as official or as reflecting the view of the US Air Force Medical Service or the US Air Force at large.

References

1. US Preventive Services Task Force Screening and interventions for childhood obesity. Guide to Clinical Preventive Services. Rockville, MD: Agency for Healthcare Research and Quality; 2005.

2. Krebs NF, Jacobson MS. and the American Academy of Pediatrics Committee on Nutrition. Prevention of pediatric overweight and obesity. Pediatrics. 2003;112:424-430.

3. Parsons TJ, Power C, Logan S, et al. Childhood predictors of adult obesity: a systematic review. Int J Obes Relat Metab Disord. 1999;23(suppl 8):S1-S107.

4. Gillman MW, Rifas-Shiman S, Berkey CS, et al. Maternal gestational diabetes, birth weight, and adolescent obesity. Pediatrics. 2003;111(3):e221-226.

5. Hediger ML, Overpeck MD, McGlynn A, et al. Growth and fatness at three to six years of age of children born small- or large-for-gestational age. Pediatrics. 1999;104:e33.-

6. He Q, Ding ZY, Fong DY, et al. Risk factors of obesity in preschool children in China: a population-based case-control study. Int J Obes Relat Metab Disord. 2000;24:1528-1536.

7. Frisancho AR. Prenatal compared with parental origins of adolescent fatness. Am J Clin Nutr. 2000;72:1186-1190.

References

1. US Preventive Services Task Force Screening and interventions for childhood obesity. Guide to Clinical Preventive Services. Rockville, MD: Agency for Healthcare Research and Quality; 2005.

2. Krebs NF, Jacobson MS. and the American Academy of Pediatrics Committee on Nutrition. Prevention of pediatric overweight and obesity. Pediatrics. 2003;112:424-430.

3. Parsons TJ, Power C, Logan S, et al. Childhood predictors of adult obesity: a systematic review. Int J Obes Relat Metab Disord. 1999;23(suppl 8):S1-S107.

4. Gillman MW, Rifas-Shiman S, Berkey CS, et al. Maternal gestational diabetes, birth weight, and adolescent obesity. Pediatrics. 2003;111(3):e221-226.

5. Hediger ML, Overpeck MD, McGlynn A, et al. Growth and fatness at three to six years of age of children born small- or large-for-gestational age. Pediatrics. 1999;104:e33.-

6. He Q, Ding ZY, Fong DY, et al. Risk factors of obesity in preschool children in China: a population-based case-control study. Int J Obes Relat Metab Disord. 2000;24:1528-1536.

7. Frisancho AR. Prenatal compared with parental origins of adolescent fatness. Am J Clin Nutr. 2000;72:1186-1190.

Issue
The Journal of Family Practice - 57(6)
Issue
The Journal of Family Practice - 57(6)
Page Number
409-410
Page Number
409-410
Publications
Publications
Topics
Article Type
Display Headline
Does birth weight predict childhood obesity?
Display Headline
Does birth weight predict childhood obesity?
Legacy Keywords
birth weight; childhood obesity; overweight; Robert K. Persons DO; Tristan L. Sevdy MD; David Krulak MD MPH
Legacy Keywords
birth weight; childhood obesity; overweight; Robert K. Persons DO; Tristan L. Sevdy MD; David Krulak MD MPH
Sections
PURLs Copyright

Evidence-based answers from the Family Physicians Inquiries Network

Disallow All Ads
Alternative CME
Article PDF Media

What’s the most effective treatment for giardiasis?

Article Type
Changed
Mon, 01/14/2019 - 11:22
Display Headline
What’s the most effective treatment for giardiasis?
EVIDENCE-BASED ANSWER

A single 2-g dose of tinidazole is the best treatment (strength of recommendation [SOR]: A, based on meta-analysis). Other drugs, such as nitazoxanide, metronidazole, mebendazole, and albendazole, can also be used (SOR: A, based on randomized controlled trial [RCT] of patient-oriented outcomes), but tinidazole has a higher clinical cure rate than these drugs. It also has a comparable side-effect profile and requires only 1 dose.

Clinical commentary

The real challenge is diagnosis
Cynthia Brown, MD
University of Nevada, Reno

As this review points out, all the available treatments for giardiasis are effective. Additional prescribing considerations include cost (500 mg metronidazole costs about 30 cents, for example, while 2 mg tinidazole costs $18) and insurance coverage. Tinidazole and metronidazole, unlike the other medications, require that the patient abstain from alcohol for 72 hours after dosing.

In my experience, the biggest challenge in treating giardiasis is deciding when to consider it in the differential and when to test for it. Presentations vary from vague symptoms such as bloating to severe diarrhea. Often the patient has not been exposed to well or stream water. You can test stool samples for ova and parasites, or serum for fluorescent antibody or enzyme-linked immunosorbent assay (ELISA).

Evidence summary

Giardia lamblia is a protozoan parasite found worldwide. Infection typically results from ingesting cysts in contaminated food or water. Patients with giardiasis may be asymptomatic or have mild to severe gastrointestinal symptoms, including explosive diarrhea, abdominal pain, steatorrhea, flatulence, bloating, nausea, and vomiting. Treatment varies widely based on geographic location, physician preference, and availability and cost of medication (TABLE).1

TABLE 1
Drugs commonly used to treat giardiasis

DRUGADULT DOSESCHEDULECOMMENT
Tinidazole2 g1 timeCan be given to children 3 years of age and older Pregnancy drug class C
Metronidazole250, 500,or 750 mg1 time or 3 times daily for 5 days. (Usually 250 mg, 3 times a day, for 5 days)Contraindicated in first trimester of pregnancy
Mebendazole100 mgTwice daily for 5 daysContraindicated in first trimester of pregnancy Pregnancy drug class B
Nitazoxanide500 mgTwice daily for 3 daysCan be given to children 1 year of age and older Available in liquid form Pregnancy drug class B
Albendazole200-400 mgTwice daily for 5 daysPregnancy drug class C
Sources: Beach M,1 and Gilbert DM et al.8
 

Tinidazole is the treatment of choice

A 2006 Cochrane Review compared 34 trials of many drug therapies for giardiasis.2 The review, which is being updated to include additional publications, evaluated both head-to-head and placebo-controlled studies, looking at dosage as well as length of drug therapy.

The review found that a single dose of tinidazole had a higher clinical cure rate than other therapies such as metronidazole (odds ratio [OR]=5.33; 95% confidence interval [CI], 2.66-10.67)2 along with a comparable side-effect profile. These findings favor tinidazole as the treatment of choice for symptomatic giardiasis.

Giardia lamblia

How effective are other drugs?

The 2006 Cochrane Review found no difference in clinical cure rate between short-term treatment (3 days) with metronidazole and longer therapy with metronidazole or other drugs. Subsequently, a single dose of metronidazole was found to be as effective as treatment for 5 days or longer (OR=0.33, 95% CI 0.08-1.34).

Since publication of the Cochrane review, several studies have further evaluated mebendazole.

  • An RCT in Cuban children 5 to 15 years of age found no difference in clinical cure rate between a 5-day course of mebendazole and more traditional therapy with quinacrine.3
  • Another RCT comparing 5 days of mebendazole with 7 days of metronidazole in 7- to 12-year-old Iranian children showed no statistical difference in microbiologic cure between the 2 regimens.4
  • Single-dose tinidazole was superior to 3 doses of mebendazole in a single day in an RCT of 122 Cuban children that measured microbiologic cure (NNT=5.5 patients with tinidazole vs mebendazole).5
 

 

 

Two RCTs found nitazoxanide to be effective (number needed to treat [NNT]=1.82) compared to placebo in adolescents and adults.6 A 3-day course of nitazoxanide was as effective as 5 days of metronidazole (80% vs 85%, P=0.61) in resolving clinical giardiasis.7

An RCT of albendazole, 400 mg for 5 days, in 28 adults found it to be as effective as 500 mg metronidazole given 3 times a day for 5 days (80% vs 83%) but less likely than metronidazole (2% vs 18%) to cause anorexia (number needed to harm [NNH]=6.25).

Recommendations

The Centers for Disease Control and Prevention recommends tinidazole, metronidazole, quinacrine, albendazole, or nitazoxanide to treat giardiasis; however, it doesn’t indicate a preference for 1 medicine over another.1 The Infectious Diseases Society of America has no guideline. The Sanford Guide to Antimicrobial Therapy recommends either a single 2-g dose of tinidazole or 500 mg of nitazoxanide PO bid for 3 days as primary treatment.8

Acknowledgments

The views expressed in this article are those of the authors and do not necessarily reflect the official policy or position of the Air Force Medical Service, nor the US Air Force.

References

1. Beach M. Prevention of specific infectious diseases—giardiasis. In: Arguin PM, Kozarsky PE, Navin AW eds. Centers for Disease Control and Prevention. Health Information for International Travel 2005-2006. Atlanta: US Department of Health and Human Services, Public Health Service; 2005. Available at: www2.ncid.cdc.gov/travel/yb/utils/ybGet.asp?section=dis&obj=giardiasis.htm. Accessed March 7, 2008.

2. Zaat JO, Mank T, Assendelft WJ. Drugs for treating giardiasis. Cochrane Database Syst Rev. 2005:CD000217.-

3. Canete R, Escobedo A, Gonzalez M, et al. Randomized clinical study of five days’ therapy with mebendazole compared to quinacrine in the treatment of symptomatic giardiasis in children. World J Gastroenterol. 2006;12:6366-6370.

4. Sadjjadi SM, Alborzi AW, Mostovfi H. Comparative clinical trial of mebendazole and metronidazole in giardiasis of children. J Trop Pediatr. 2001;47:176-178.

5. Canete R, Escobedo A, Gonzalez M, et al. A randomized, controlled, open-label trial of a single day of mebendazole versus a single dose of tinidazole in the treatment of giardiasis in children. Curr Med Res Opin. 2006;22:2131-2136.

6. Rossignol JF, Ayoub A, Ayers MS, et al. Treatment of diarrhea caused by Giardia intestinalis and Entameba histolytica or E dispar: A Randomized, double-blind, placebo-controlled study of nitazoxanide. J Infect Dis. 2001;184:381-384.

7. Ortiz JJ, Ayoub A, Gargala G, et al. Randomized clinical study of nitazoxanide compared to metronidazole in the treatment of symptomatic giardiasis in children from northern Peru. Aliment Pharmacol Ther. 2001;15:1409-1415.

8. Gilbert DM, Eliopoulos GM, Moellering RC, et al. The Sanford Guide to Antimicrobial Therapy 2006. 36th ed. Sperryville, Va: Antimicrobial Therapy; 2006:95.

Article PDF
Author and Disclosure Information

Jeffrey D. Kiser, MD
Christopher P. Paulson, MD, FAAFP
William Nichols, MLS
Eglin AFB Family Medicine Residency, Eglin AFB, Fla

Issue
The Journal of Family Practice - 57(4)
Publications
Topics
Page Number
270-271
Legacy Keywords
giardiasis; Giardia lamblia; protozoan parasite; Jeffrey D. Kiser MD; Christopher P. Paulson MD; Cynthia Brown MD; Family Physicians Inquiries Network
Sections
Author and Disclosure Information

Jeffrey D. Kiser, MD
Christopher P. Paulson, MD, FAAFP
William Nichols, MLS
Eglin AFB Family Medicine Residency, Eglin AFB, Fla

Author and Disclosure Information

Jeffrey D. Kiser, MD
Christopher P. Paulson, MD, FAAFP
William Nichols, MLS
Eglin AFB Family Medicine Residency, Eglin AFB, Fla

Article PDF
Article PDF
EVIDENCE-BASED ANSWER

A single 2-g dose of tinidazole is the best treatment (strength of recommendation [SOR]: A, based on meta-analysis). Other drugs, such as nitazoxanide, metronidazole, mebendazole, and albendazole, can also be used (SOR: A, based on randomized controlled trial [RCT] of patient-oriented outcomes), but tinidazole has a higher clinical cure rate than these drugs. It also has a comparable side-effect profile and requires only 1 dose.

Clinical commentary

The real challenge is diagnosis
Cynthia Brown, MD
University of Nevada, Reno

As this review points out, all the available treatments for giardiasis are effective. Additional prescribing considerations include cost (500 mg metronidazole costs about 30 cents, for example, while 2 mg tinidazole costs $18) and insurance coverage. Tinidazole and metronidazole, unlike the other medications, require that the patient abstain from alcohol for 72 hours after dosing.

In my experience, the biggest challenge in treating giardiasis is deciding when to consider it in the differential and when to test for it. Presentations vary from vague symptoms such as bloating to severe diarrhea. Often the patient has not been exposed to well or stream water. You can test stool samples for ova and parasites, or serum for fluorescent antibody or enzyme-linked immunosorbent assay (ELISA).

Evidence summary

Giardia lamblia is a protozoan parasite found worldwide. Infection typically results from ingesting cysts in contaminated food or water. Patients with giardiasis may be asymptomatic or have mild to severe gastrointestinal symptoms, including explosive diarrhea, abdominal pain, steatorrhea, flatulence, bloating, nausea, and vomiting. Treatment varies widely based on geographic location, physician preference, and availability and cost of medication (TABLE).1

TABLE 1
Drugs commonly used to treat giardiasis

DRUGADULT DOSESCHEDULECOMMENT
Tinidazole2 g1 timeCan be given to children 3 years of age and older Pregnancy drug class C
Metronidazole250, 500,or 750 mg1 time or 3 times daily for 5 days. (Usually 250 mg, 3 times a day, for 5 days)Contraindicated in first trimester of pregnancy
Mebendazole100 mgTwice daily for 5 daysContraindicated in first trimester of pregnancy Pregnancy drug class B
Nitazoxanide500 mgTwice daily for 3 daysCan be given to children 1 year of age and older Available in liquid form Pregnancy drug class B
Albendazole200-400 mgTwice daily for 5 daysPregnancy drug class C
Sources: Beach M,1 and Gilbert DM et al.8
 

Tinidazole is the treatment of choice

A 2006 Cochrane Review compared 34 trials of many drug therapies for giardiasis.2 The review, which is being updated to include additional publications, evaluated both head-to-head and placebo-controlled studies, looking at dosage as well as length of drug therapy.

The review found that a single dose of tinidazole had a higher clinical cure rate than other therapies such as metronidazole (odds ratio [OR]=5.33; 95% confidence interval [CI], 2.66-10.67)2 along with a comparable side-effect profile. These findings favor tinidazole as the treatment of choice for symptomatic giardiasis.

Giardia lamblia

How effective are other drugs?

The 2006 Cochrane Review found no difference in clinical cure rate between short-term treatment (3 days) with metronidazole and longer therapy with metronidazole or other drugs. Subsequently, a single dose of metronidazole was found to be as effective as treatment for 5 days or longer (OR=0.33, 95% CI 0.08-1.34).

Since publication of the Cochrane review, several studies have further evaluated mebendazole.

  • An RCT in Cuban children 5 to 15 years of age found no difference in clinical cure rate between a 5-day course of mebendazole and more traditional therapy with quinacrine.3
  • Another RCT comparing 5 days of mebendazole with 7 days of metronidazole in 7- to 12-year-old Iranian children showed no statistical difference in microbiologic cure between the 2 regimens.4
  • Single-dose tinidazole was superior to 3 doses of mebendazole in a single day in an RCT of 122 Cuban children that measured microbiologic cure (NNT=5.5 patients with tinidazole vs mebendazole).5
 

 

 

Two RCTs found nitazoxanide to be effective (number needed to treat [NNT]=1.82) compared to placebo in adolescents and adults.6 A 3-day course of nitazoxanide was as effective as 5 days of metronidazole (80% vs 85%, P=0.61) in resolving clinical giardiasis.7

An RCT of albendazole, 400 mg for 5 days, in 28 adults found it to be as effective as 500 mg metronidazole given 3 times a day for 5 days (80% vs 83%) but less likely than metronidazole (2% vs 18%) to cause anorexia (number needed to harm [NNH]=6.25).

Recommendations

The Centers for Disease Control and Prevention recommends tinidazole, metronidazole, quinacrine, albendazole, or nitazoxanide to treat giardiasis; however, it doesn’t indicate a preference for 1 medicine over another.1 The Infectious Diseases Society of America has no guideline. The Sanford Guide to Antimicrobial Therapy recommends either a single 2-g dose of tinidazole or 500 mg of nitazoxanide PO bid for 3 days as primary treatment.8

Acknowledgments

The views expressed in this article are those of the authors and do not necessarily reflect the official policy or position of the Air Force Medical Service, nor the US Air Force.

EVIDENCE-BASED ANSWER

A single 2-g dose of tinidazole is the best treatment (strength of recommendation [SOR]: A, based on meta-analysis). Other drugs, such as nitazoxanide, metronidazole, mebendazole, and albendazole, can also be used (SOR: A, based on randomized controlled trial [RCT] of patient-oriented outcomes), but tinidazole has a higher clinical cure rate than these drugs. It also has a comparable side-effect profile and requires only 1 dose.

Clinical commentary

The real challenge is diagnosis
Cynthia Brown, MD
University of Nevada, Reno

As this review points out, all the available treatments for giardiasis are effective. Additional prescribing considerations include cost (500 mg metronidazole costs about 30 cents, for example, while 2 mg tinidazole costs $18) and insurance coverage. Tinidazole and metronidazole, unlike the other medications, require that the patient abstain from alcohol for 72 hours after dosing.

In my experience, the biggest challenge in treating giardiasis is deciding when to consider it in the differential and when to test for it. Presentations vary from vague symptoms such as bloating to severe diarrhea. Often the patient has not been exposed to well or stream water. You can test stool samples for ova and parasites, or serum for fluorescent antibody or enzyme-linked immunosorbent assay (ELISA).

Evidence summary

Giardia lamblia is a protozoan parasite found worldwide. Infection typically results from ingesting cysts in contaminated food or water. Patients with giardiasis may be asymptomatic or have mild to severe gastrointestinal symptoms, including explosive diarrhea, abdominal pain, steatorrhea, flatulence, bloating, nausea, and vomiting. Treatment varies widely based on geographic location, physician preference, and availability and cost of medication (TABLE).1

TABLE 1
Drugs commonly used to treat giardiasis

DRUGADULT DOSESCHEDULECOMMENT
Tinidazole2 g1 timeCan be given to children 3 years of age and older Pregnancy drug class C
Metronidazole250, 500,or 750 mg1 time or 3 times daily for 5 days. (Usually 250 mg, 3 times a day, for 5 days)Contraindicated in first trimester of pregnancy
Mebendazole100 mgTwice daily for 5 daysContraindicated in first trimester of pregnancy Pregnancy drug class B
Nitazoxanide500 mgTwice daily for 3 daysCan be given to children 1 year of age and older Available in liquid form Pregnancy drug class B
Albendazole200-400 mgTwice daily for 5 daysPregnancy drug class C
Sources: Beach M,1 and Gilbert DM et al.8
 

Tinidazole is the treatment of choice

A 2006 Cochrane Review compared 34 trials of many drug therapies for giardiasis.2 The review, which is being updated to include additional publications, evaluated both head-to-head and placebo-controlled studies, looking at dosage as well as length of drug therapy.

The review found that a single dose of tinidazole had a higher clinical cure rate than other therapies such as metronidazole (odds ratio [OR]=5.33; 95% confidence interval [CI], 2.66-10.67)2 along with a comparable side-effect profile. These findings favor tinidazole as the treatment of choice for symptomatic giardiasis.

Giardia lamblia

How effective are other drugs?

The 2006 Cochrane Review found no difference in clinical cure rate between short-term treatment (3 days) with metronidazole and longer therapy with metronidazole or other drugs. Subsequently, a single dose of metronidazole was found to be as effective as treatment for 5 days or longer (OR=0.33, 95% CI 0.08-1.34).

Since publication of the Cochrane review, several studies have further evaluated mebendazole.

  • An RCT in Cuban children 5 to 15 years of age found no difference in clinical cure rate between a 5-day course of mebendazole and more traditional therapy with quinacrine.3
  • Another RCT comparing 5 days of mebendazole with 7 days of metronidazole in 7- to 12-year-old Iranian children showed no statistical difference in microbiologic cure between the 2 regimens.4
  • Single-dose tinidazole was superior to 3 doses of mebendazole in a single day in an RCT of 122 Cuban children that measured microbiologic cure (NNT=5.5 patients with tinidazole vs mebendazole).5
 

 

 

Two RCTs found nitazoxanide to be effective (number needed to treat [NNT]=1.82) compared to placebo in adolescents and adults.6 A 3-day course of nitazoxanide was as effective as 5 days of metronidazole (80% vs 85%, P=0.61) in resolving clinical giardiasis.7

An RCT of albendazole, 400 mg for 5 days, in 28 adults found it to be as effective as 500 mg metronidazole given 3 times a day for 5 days (80% vs 83%) but less likely than metronidazole (2% vs 18%) to cause anorexia (number needed to harm [NNH]=6.25).

Recommendations

The Centers for Disease Control and Prevention recommends tinidazole, metronidazole, quinacrine, albendazole, or nitazoxanide to treat giardiasis; however, it doesn’t indicate a preference for 1 medicine over another.1 The Infectious Diseases Society of America has no guideline. The Sanford Guide to Antimicrobial Therapy recommends either a single 2-g dose of tinidazole or 500 mg of nitazoxanide PO bid for 3 days as primary treatment.8

Acknowledgments

The views expressed in this article are those of the authors and do not necessarily reflect the official policy or position of the Air Force Medical Service, nor the US Air Force.

References

1. Beach M. Prevention of specific infectious diseases—giardiasis. In: Arguin PM, Kozarsky PE, Navin AW eds. Centers for Disease Control and Prevention. Health Information for International Travel 2005-2006. Atlanta: US Department of Health and Human Services, Public Health Service; 2005. Available at: www2.ncid.cdc.gov/travel/yb/utils/ybGet.asp?section=dis&obj=giardiasis.htm. Accessed March 7, 2008.

2. Zaat JO, Mank T, Assendelft WJ. Drugs for treating giardiasis. Cochrane Database Syst Rev. 2005:CD000217.-

3. Canete R, Escobedo A, Gonzalez M, et al. Randomized clinical study of five days’ therapy with mebendazole compared to quinacrine in the treatment of symptomatic giardiasis in children. World J Gastroenterol. 2006;12:6366-6370.

4. Sadjjadi SM, Alborzi AW, Mostovfi H. Comparative clinical trial of mebendazole and metronidazole in giardiasis of children. J Trop Pediatr. 2001;47:176-178.

5. Canete R, Escobedo A, Gonzalez M, et al. A randomized, controlled, open-label trial of a single day of mebendazole versus a single dose of tinidazole in the treatment of giardiasis in children. Curr Med Res Opin. 2006;22:2131-2136.

6. Rossignol JF, Ayoub A, Ayers MS, et al. Treatment of diarrhea caused by Giardia intestinalis and Entameba histolytica or E dispar: A Randomized, double-blind, placebo-controlled study of nitazoxanide. J Infect Dis. 2001;184:381-384.

7. Ortiz JJ, Ayoub A, Gargala G, et al. Randomized clinical study of nitazoxanide compared to metronidazole in the treatment of symptomatic giardiasis in children from northern Peru. Aliment Pharmacol Ther. 2001;15:1409-1415.

8. Gilbert DM, Eliopoulos GM, Moellering RC, et al. The Sanford Guide to Antimicrobial Therapy 2006. 36th ed. Sperryville, Va: Antimicrobial Therapy; 2006:95.

References

1. Beach M. Prevention of specific infectious diseases—giardiasis. In: Arguin PM, Kozarsky PE, Navin AW eds. Centers for Disease Control and Prevention. Health Information for International Travel 2005-2006. Atlanta: US Department of Health and Human Services, Public Health Service; 2005. Available at: www2.ncid.cdc.gov/travel/yb/utils/ybGet.asp?section=dis&obj=giardiasis.htm. Accessed March 7, 2008.

2. Zaat JO, Mank T, Assendelft WJ. Drugs for treating giardiasis. Cochrane Database Syst Rev. 2005:CD000217.-

3. Canete R, Escobedo A, Gonzalez M, et al. Randomized clinical study of five days’ therapy with mebendazole compared to quinacrine in the treatment of symptomatic giardiasis in children. World J Gastroenterol. 2006;12:6366-6370.

4. Sadjjadi SM, Alborzi AW, Mostovfi H. Comparative clinical trial of mebendazole and metronidazole in giardiasis of children. J Trop Pediatr. 2001;47:176-178.

5. Canete R, Escobedo A, Gonzalez M, et al. A randomized, controlled, open-label trial of a single day of mebendazole versus a single dose of tinidazole in the treatment of giardiasis in children. Curr Med Res Opin. 2006;22:2131-2136.

6. Rossignol JF, Ayoub A, Ayers MS, et al. Treatment of diarrhea caused by Giardia intestinalis and Entameba histolytica or E dispar: A Randomized, double-blind, placebo-controlled study of nitazoxanide. J Infect Dis. 2001;184:381-384.

7. Ortiz JJ, Ayoub A, Gargala G, et al. Randomized clinical study of nitazoxanide compared to metronidazole in the treatment of symptomatic giardiasis in children from northern Peru. Aliment Pharmacol Ther. 2001;15:1409-1415.

8. Gilbert DM, Eliopoulos GM, Moellering RC, et al. The Sanford Guide to Antimicrobial Therapy 2006. 36th ed. Sperryville, Va: Antimicrobial Therapy; 2006:95.

Issue
The Journal of Family Practice - 57(4)
Issue
The Journal of Family Practice - 57(4)
Page Number
270-271
Page Number
270-271
Publications
Publications
Topics
Article Type
Display Headline
What’s the most effective treatment for giardiasis?
Display Headline
What’s the most effective treatment for giardiasis?
Legacy Keywords
giardiasis; Giardia lamblia; protozoan parasite; Jeffrey D. Kiser MD; Christopher P. Paulson MD; Cynthia Brown MD; Family Physicians Inquiries Network
Legacy Keywords
giardiasis; Giardia lamblia; protozoan parasite; Jeffrey D. Kiser MD; Christopher P. Paulson MD; Cynthia Brown MD; Family Physicians Inquiries Network
Sections
PURLs Copyright

Evidence-based answers from the Family Physicians Inquiries Network

Disallow All Ads
Article PDF Media

When should travelers begin malaria prophylaxis?

Article Type
Changed
Mon, 01/14/2019 - 11:21
Display Headline
When should travelers begin malaria prophylaxis?
EVIDENCE-BASED ANSWER

Travelers should start on chloroquine 1 to 2 weeks before entering an area without chloroquine resistance (strength of recommendation [SOR]: C, based on expert opinion). In areas with chloroquine-resistant Plasmodium falciparum, travelers will need to take atovaquone/proguanil, doxycycline, or primaquine 1 day before entering the area, or mefloquine 2 to 7 weeks before travel (SOR: B, based on prospective patient-oriented outcomes and expert opinion).

Before prescribing medications, determine malaria risk and sensitivity of Plasmodium species by country at wwwn.cdc.gov/travel/yellowBookCh5MalariaYellowFeverTable.aspx (SOR: C, based on patient-oriented expert opinion).

Clinical commentary

5 tips to help travelers avoid malaria
Brian V. Reamy, MD
Uniformed Services University, Bethesda, Md

Despite our best efforts, more than 10,000 American and European travelers contract malaria each year. Five clinical pointers are helpful in prescribing malaria prophylaxis and preventing malaria in travelers.

1. Advise patients that they’ll need to get their antimalarials before they leave for their trip. The CDC recommends against the purchase of antimalarials while overseas because of concerns about product quality.

2. Encourage patients to plan ahead. Most local community pharmacies do not routinely stock antimalarials and must special order them. If a patient mentions an upcoming trip, advise them that they’ll need to allow an extra 2 weeks to obtain their medications.

3. Consult 1 of 2 continuously updated Web sites prior to selecting a medication for malaria prophylaxis: wwwn.cdc.gov/travel/destinationList.aspx or www.who.int/ith/en.

Start times vary from 1 day to several weeks prior to travel based on the medication selected.

4. Encourage patients to spray clothing with permethrin prior to travel. Permethrin remains effective as a repellent even after months of clothing use and multiple washes.

5. Encourage travelers to finish their medication after they return and to report unexplained fevers for up to 1 year after travel.

Evidence summary

 

Travelers to malaria-endemic areas should avoid mosquito bites by using netting and repellents, and use chemoprophylaxis to prevent infection.

Although no drug regimen guarantees protection against malaria, physicians should prescribe 1 of several options based on the location of travel, the susceptibility of indigenous P falciparum, and the side-effect profile.1

Timing and dosage of prophylactic drugs

Prophylactic medications must be started at different times before travel, but for some medications the optimal time to initiate treatment is unclear. Evidence-based recommendations2,3 with consideration for side-effect profiles are given in the TABLE.

In contrast to the pretreatment times for all other malarial prophylaxes, the generally accepted pretreatment time for mefloquine is 1 to 2 weeks before entering a risk area. However, this may still be inadequate due to the drug’s long half-life, which results in a long delay in reaching therapeutic blood levels.4 The evidence indicates that mefloquine should be started at least 2, and as many as 7, weeks before travel.

The standard recommended dose of 250 mg/week of mefloquine “produces maximum steady-state plasma concentrations of 1000 to 2000 mcg/L, which are reached only after 7 to 10 weeks.”4 One study of 293 children under the age of 5 years in Malawi found that plasma concentrations of mefloquine were below prophylactic level (500 mcg/mL) against P falciparum until the fourth to seventh week of once-weekly dosing (P<.0003).5

One way of reaching prophylactic levels earlier would be to give mefloquine 250 mg daily for 3 days followed by 250 mg weekly.4 A safety study of 157 healthy US Marine volunteers showed that preloading achieves prophylactic blood levels of mefloquine by the third day while weekly mefloquine is subprophylactic until the fifth week.4

 

 

While a study of the long-term use of mefloquine in 421 healthy Peace Corps volunteers has shown it to be safe,6 clinical trials and case reports indicate that a loading dose of mefloquine is associated with adverse drug events, which include neuropsychiatric and gastrointestinal symptoms.4,7

TABLE
Evidence-based recommendations for prevention of malaria2-3,8

DRUGUSAGEADULT DOSETREATMENT SCHEDULE
Atovaquone/proguanil Contraindicated in pregnancyProphylaxis in areas with chloroquine-resistant or mefloquine-resistant P falciparum1 tablet orally each day 250 mg atovaquone and 100 mg proguanil hydrochloride)Daily from 1 day prior to entry until 7 days after leaving
ChloroquineProphylaxis only in areas with chloroquine-sensitive P falciparum300 mg base (500 mg salt) orally, once/weekWeekly from 2 weeks prior to entry until 4 weeks after leaving (take on the same day of the week)
Doxycycline Contraindicated in children <8 years of age and pregnant womenProphylaxis in areas with chloroquine-resistant or mefloquine-resistant P falciparum100 mg orally, dailyDaily from 1 day prior to entry until 4 weeks after leaving
MefloquineProphylaxis in areas with chloroquine-resistant P falciparum228 mg base (250 mg salt) orally, once/weekWeekly from 2–7 weeks before entry until 4 weeks after leaving (take on the same day of the week)
PrimaquineAn option for prophylaxis in special circumstances30 mg base (52.6 mg salt) orally, dailyDaily from 1 day prior to entry until 7 days after leaving
 

Recommendations from others

The World Health Organization (WHO) states that “weekly mefloquine should be started at least 1 week, but preferably 2–3 weeks before departure, to achieve higher pre-travel blood levels and to allow side effects to be detected before travel so that possible alternatives can be considered.”8

Centers for Disease Control and Prevention recommendations integrate recommendations from WHO and Cochrane.

Acknowledgments

The opinions and assertions contained herein are the private views of the authors and not to be construed as official, or as reflecting the views of the US Air Force Medical Service or the US Air Force at large.

References

1. Chen LH, Keystone JS. New strategies for the prevention of malaria in travelers. Infect Dis Clin North Am 2005;19:185-210.

2. Physicians’ Desk Reference. 61st ed. Montvale, NJ: Thomson; 2007:2786.

3. Parise M, Barber A, Mali S. Prevention of specific infectious diseases—malaria. In Arguin PM, Kozarsky PE, Navin AW (eds), Health Information for International Travel 2005-2006. Atlanta, Ga: US Department of Health and Human Services, Public Health Service, Centers for Disease Control and Prevention; 2007. Available at: wwwn.cdc.gov/travel/yellowBookCh4-Malaria.aspx. Accessed on October 11, 2007.

4. Boudreau E, Schuster B, Sanchez J, et al. Tolerability of prophylactic Lariam regimens. Trop Med Parasitol 1993;44:257-265.

5. Slutsker LM, Khoromana CO, Payne D, et al. Mefloquine therapy for Plasmodium falciparum. malaria in children under 5 years of age in Malawi: in vivo/in vitro efficacy and correlation of drug concentration with parasitological outcome. Bull World Health Organ 1990;68:53-59.

6. Lobel HO, Miani M, Eng T, et al. Long-term malaria prophylaxis with weekly mefloquine. Lancet 1993;341:848-51.

7. Schlagenhauf P. Mefloquine for malaria chemoprophylaxis 1992-1998: a review. J Travel Med 1999;6:122-133.

8. International Travel and Health 2005. Chapter 7: Malaria. Geneva: World Health Organization; 2005. Available at: whqlibdoc.who.int/publications/2005/9241580364_chap7.pdf. Accessed on October 11, 2007.

Article PDF
Author and Disclosure Information

Steven L. Clark, MD
Paul Crawford, MD
Eglin Air Force Base Family Medicine Residency, Eglin Air Force Base, Fla

William Nichols, MLS
Librarian, Eglin Regional Hospital, Eglin Air Force Base, Fla

Issue
The Journal of Family Practice - 56(11)
Publications
Topics
Page Number
950-951
Legacy Keywords
Steven L. Clark;MD; Paul Crawford;MD; William Nichols;MLS; Brian V. Reamy ;MD; malaria; prophylaxis; anopheles; chloroquine; resistant; Plasmodium falciparum; travel; atovaquone; proguanil; doxycycline; primaquine; mefloquine; schedule; infections; parasite; CDC; WHO
Sections
Author and Disclosure Information

Steven L. Clark, MD
Paul Crawford, MD
Eglin Air Force Base Family Medicine Residency, Eglin Air Force Base, Fla

William Nichols, MLS
Librarian, Eglin Regional Hospital, Eglin Air Force Base, Fla

Author and Disclosure Information

Steven L. Clark, MD
Paul Crawford, MD
Eglin Air Force Base Family Medicine Residency, Eglin Air Force Base, Fla

William Nichols, MLS
Librarian, Eglin Regional Hospital, Eglin Air Force Base, Fla

Article PDF
Article PDF
EVIDENCE-BASED ANSWER

Travelers should start on chloroquine 1 to 2 weeks before entering an area without chloroquine resistance (strength of recommendation [SOR]: C, based on expert opinion). In areas with chloroquine-resistant Plasmodium falciparum, travelers will need to take atovaquone/proguanil, doxycycline, or primaquine 1 day before entering the area, or mefloquine 2 to 7 weeks before travel (SOR: B, based on prospective patient-oriented outcomes and expert opinion).

Before prescribing medications, determine malaria risk and sensitivity of Plasmodium species by country at wwwn.cdc.gov/travel/yellowBookCh5MalariaYellowFeverTable.aspx (SOR: C, based on patient-oriented expert opinion).

Clinical commentary

5 tips to help travelers avoid malaria
Brian V. Reamy, MD
Uniformed Services University, Bethesda, Md

Despite our best efforts, more than 10,000 American and European travelers contract malaria each year. Five clinical pointers are helpful in prescribing malaria prophylaxis and preventing malaria in travelers.

1. Advise patients that they’ll need to get their antimalarials before they leave for their trip. The CDC recommends against the purchase of antimalarials while overseas because of concerns about product quality.

2. Encourage patients to plan ahead. Most local community pharmacies do not routinely stock antimalarials and must special order them. If a patient mentions an upcoming trip, advise them that they’ll need to allow an extra 2 weeks to obtain their medications.

3. Consult 1 of 2 continuously updated Web sites prior to selecting a medication for malaria prophylaxis: wwwn.cdc.gov/travel/destinationList.aspx or www.who.int/ith/en.

Start times vary from 1 day to several weeks prior to travel based on the medication selected.

4. Encourage patients to spray clothing with permethrin prior to travel. Permethrin remains effective as a repellent even after months of clothing use and multiple washes.

5. Encourage travelers to finish their medication after they return and to report unexplained fevers for up to 1 year after travel.

Evidence summary

 

Travelers to malaria-endemic areas should avoid mosquito bites by using netting and repellents, and use chemoprophylaxis to prevent infection.

Although no drug regimen guarantees protection against malaria, physicians should prescribe 1 of several options based on the location of travel, the susceptibility of indigenous P falciparum, and the side-effect profile.1

Timing and dosage of prophylactic drugs

Prophylactic medications must be started at different times before travel, but for some medications the optimal time to initiate treatment is unclear. Evidence-based recommendations2,3 with consideration for side-effect profiles are given in the TABLE.

In contrast to the pretreatment times for all other malarial prophylaxes, the generally accepted pretreatment time for mefloquine is 1 to 2 weeks before entering a risk area. However, this may still be inadequate due to the drug’s long half-life, which results in a long delay in reaching therapeutic blood levels.4 The evidence indicates that mefloquine should be started at least 2, and as many as 7, weeks before travel.

The standard recommended dose of 250 mg/week of mefloquine “produces maximum steady-state plasma concentrations of 1000 to 2000 mcg/L, which are reached only after 7 to 10 weeks.”4 One study of 293 children under the age of 5 years in Malawi found that plasma concentrations of mefloquine were below prophylactic level (500 mcg/mL) against P falciparum until the fourth to seventh week of once-weekly dosing (P<.0003).5

One way of reaching prophylactic levels earlier would be to give mefloquine 250 mg daily for 3 days followed by 250 mg weekly.4 A safety study of 157 healthy US Marine volunteers showed that preloading achieves prophylactic blood levels of mefloquine by the third day while weekly mefloquine is subprophylactic until the fifth week.4

 

 

While a study of the long-term use of mefloquine in 421 healthy Peace Corps volunteers has shown it to be safe,6 clinical trials and case reports indicate that a loading dose of mefloquine is associated with adverse drug events, which include neuropsychiatric and gastrointestinal symptoms.4,7

TABLE
Evidence-based recommendations for prevention of malaria2-3,8

DRUGUSAGEADULT DOSETREATMENT SCHEDULE
Atovaquone/proguanil Contraindicated in pregnancyProphylaxis in areas with chloroquine-resistant or mefloquine-resistant P falciparum1 tablet orally each day 250 mg atovaquone and 100 mg proguanil hydrochloride)Daily from 1 day prior to entry until 7 days after leaving
ChloroquineProphylaxis only in areas with chloroquine-sensitive P falciparum300 mg base (500 mg salt) orally, once/weekWeekly from 2 weeks prior to entry until 4 weeks after leaving (take on the same day of the week)
Doxycycline Contraindicated in children <8 years of age and pregnant womenProphylaxis in areas with chloroquine-resistant or mefloquine-resistant P falciparum100 mg orally, dailyDaily from 1 day prior to entry until 4 weeks after leaving
MefloquineProphylaxis in areas with chloroquine-resistant P falciparum228 mg base (250 mg salt) orally, once/weekWeekly from 2–7 weeks before entry until 4 weeks after leaving (take on the same day of the week)
PrimaquineAn option for prophylaxis in special circumstances30 mg base (52.6 mg salt) orally, dailyDaily from 1 day prior to entry until 7 days after leaving
 

Recommendations from others

The World Health Organization (WHO) states that “weekly mefloquine should be started at least 1 week, but preferably 2–3 weeks before departure, to achieve higher pre-travel blood levels and to allow side effects to be detected before travel so that possible alternatives can be considered.”8

Centers for Disease Control and Prevention recommendations integrate recommendations from WHO and Cochrane.

Acknowledgments

The opinions and assertions contained herein are the private views of the authors and not to be construed as official, or as reflecting the views of the US Air Force Medical Service or the US Air Force at large.

EVIDENCE-BASED ANSWER

Travelers should start on chloroquine 1 to 2 weeks before entering an area without chloroquine resistance (strength of recommendation [SOR]: C, based on expert opinion). In areas with chloroquine-resistant Plasmodium falciparum, travelers will need to take atovaquone/proguanil, doxycycline, or primaquine 1 day before entering the area, or mefloquine 2 to 7 weeks before travel (SOR: B, based on prospective patient-oriented outcomes and expert opinion).

Before prescribing medications, determine malaria risk and sensitivity of Plasmodium species by country at wwwn.cdc.gov/travel/yellowBookCh5MalariaYellowFeverTable.aspx (SOR: C, based on patient-oriented expert opinion).

Clinical commentary

5 tips to help travelers avoid malaria
Brian V. Reamy, MD
Uniformed Services University, Bethesda, Md

Despite our best efforts, more than 10,000 American and European travelers contract malaria each year. Five clinical pointers are helpful in prescribing malaria prophylaxis and preventing malaria in travelers.

1. Advise patients that they’ll need to get their antimalarials before they leave for their trip. The CDC recommends against the purchase of antimalarials while overseas because of concerns about product quality.

2. Encourage patients to plan ahead. Most local community pharmacies do not routinely stock antimalarials and must special order them. If a patient mentions an upcoming trip, advise them that they’ll need to allow an extra 2 weeks to obtain their medications.

3. Consult 1 of 2 continuously updated Web sites prior to selecting a medication for malaria prophylaxis: wwwn.cdc.gov/travel/destinationList.aspx or www.who.int/ith/en.

Start times vary from 1 day to several weeks prior to travel based on the medication selected.

4. Encourage patients to spray clothing with permethrin prior to travel. Permethrin remains effective as a repellent even after months of clothing use and multiple washes.

5. Encourage travelers to finish their medication after they return and to report unexplained fevers for up to 1 year after travel.

Evidence summary

 

Travelers to malaria-endemic areas should avoid mosquito bites by using netting and repellents, and use chemoprophylaxis to prevent infection.

Although no drug regimen guarantees protection against malaria, physicians should prescribe 1 of several options based on the location of travel, the susceptibility of indigenous P falciparum, and the side-effect profile.1

Timing and dosage of prophylactic drugs

Prophylactic medications must be started at different times before travel, but for some medications the optimal time to initiate treatment is unclear. Evidence-based recommendations2,3 with consideration for side-effect profiles are given in the TABLE.

In contrast to the pretreatment times for all other malarial prophylaxes, the generally accepted pretreatment time for mefloquine is 1 to 2 weeks before entering a risk area. However, this may still be inadequate due to the drug’s long half-life, which results in a long delay in reaching therapeutic blood levels.4 The evidence indicates that mefloquine should be started at least 2, and as many as 7, weeks before travel.

The standard recommended dose of 250 mg/week of mefloquine “produces maximum steady-state plasma concentrations of 1000 to 2000 mcg/L, which are reached only after 7 to 10 weeks.”4 One study of 293 children under the age of 5 years in Malawi found that plasma concentrations of mefloquine were below prophylactic level (500 mcg/mL) against P falciparum until the fourth to seventh week of once-weekly dosing (P<.0003).5

One way of reaching prophylactic levels earlier would be to give mefloquine 250 mg daily for 3 days followed by 250 mg weekly.4 A safety study of 157 healthy US Marine volunteers showed that preloading achieves prophylactic blood levels of mefloquine by the third day while weekly mefloquine is subprophylactic until the fifth week.4

 

 

While a study of the long-term use of mefloquine in 421 healthy Peace Corps volunteers has shown it to be safe,6 clinical trials and case reports indicate that a loading dose of mefloquine is associated with adverse drug events, which include neuropsychiatric and gastrointestinal symptoms.4,7

TABLE
Evidence-based recommendations for prevention of malaria2-3,8

DRUGUSAGEADULT DOSETREATMENT SCHEDULE
Atovaquone/proguanil Contraindicated in pregnancyProphylaxis in areas with chloroquine-resistant or mefloquine-resistant P falciparum1 tablet orally each day 250 mg atovaquone and 100 mg proguanil hydrochloride)Daily from 1 day prior to entry until 7 days after leaving
ChloroquineProphylaxis only in areas with chloroquine-sensitive P falciparum300 mg base (500 mg salt) orally, once/weekWeekly from 2 weeks prior to entry until 4 weeks after leaving (take on the same day of the week)
Doxycycline Contraindicated in children <8 years of age and pregnant womenProphylaxis in areas with chloroquine-resistant or mefloquine-resistant P falciparum100 mg orally, dailyDaily from 1 day prior to entry until 4 weeks after leaving
MefloquineProphylaxis in areas with chloroquine-resistant P falciparum228 mg base (250 mg salt) orally, once/weekWeekly from 2–7 weeks before entry until 4 weeks after leaving (take on the same day of the week)
PrimaquineAn option for prophylaxis in special circumstances30 mg base (52.6 mg salt) orally, dailyDaily from 1 day prior to entry until 7 days after leaving
 

Recommendations from others

The World Health Organization (WHO) states that “weekly mefloquine should be started at least 1 week, but preferably 2–3 weeks before departure, to achieve higher pre-travel blood levels and to allow side effects to be detected before travel so that possible alternatives can be considered.”8

Centers for Disease Control and Prevention recommendations integrate recommendations from WHO and Cochrane.

Acknowledgments

The opinions and assertions contained herein are the private views of the authors and not to be construed as official, or as reflecting the views of the US Air Force Medical Service or the US Air Force at large.

References

1. Chen LH, Keystone JS. New strategies for the prevention of malaria in travelers. Infect Dis Clin North Am 2005;19:185-210.

2. Physicians’ Desk Reference. 61st ed. Montvale, NJ: Thomson; 2007:2786.

3. Parise M, Barber A, Mali S. Prevention of specific infectious diseases—malaria. In Arguin PM, Kozarsky PE, Navin AW (eds), Health Information for International Travel 2005-2006. Atlanta, Ga: US Department of Health and Human Services, Public Health Service, Centers for Disease Control and Prevention; 2007. Available at: wwwn.cdc.gov/travel/yellowBookCh4-Malaria.aspx. Accessed on October 11, 2007.

4. Boudreau E, Schuster B, Sanchez J, et al. Tolerability of prophylactic Lariam regimens. Trop Med Parasitol 1993;44:257-265.

5. Slutsker LM, Khoromana CO, Payne D, et al. Mefloquine therapy for Plasmodium falciparum. malaria in children under 5 years of age in Malawi: in vivo/in vitro efficacy and correlation of drug concentration with parasitological outcome. Bull World Health Organ 1990;68:53-59.

6. Lobel HO, Miani M, Eng T, et al. Long-term malaria prophylaxis with weekly mefloquine. Lancet 1993;341:848-51.

7. Schlagenhauf P. Mefloquine for malaria chemoprophylaxis 1992-1998: a review. J Travel Med 1999;6:122-133.

8. International Travel and Health 2005. Chapter 7: Malaria. Geneva: World Health Organization; 2005. Available at: whqlibdoc.who.int/publications/2005/9241580364_chap7.pdf. Accessed on October 11, 2007.

References

1. Chen LH, Keystone JS. New strategies for the prevention of malaria in travelers. Infect Dis Clin North Am 2005;19:185-210.

2. Physicians’ Desk Reference. 61st ed. Montvale, NJ: Thomson; 2007:2786.

3. Parise M, Barber A, Mali S. Prevention of specific infectious diseases—malaria. In Arguin PM, Kozarsky PE, Navin AW (eds), Health Information for International Travel 2005-2006. Atlanta, Ga: US Department of Health and Human Services, Public Health Service, Centers for Disease Control and Prevention; 2007. Available at: wwwn.cdc.gov/travel/yellowBookCh4-Malaria.aspx. Accessed on October 11, 2007.

4. Boudreau E, Schuster B, Sanchez J, et al. Tolerability of prophylactic Lariam regimens. Trop Med Parasitol 1993;44:257-265.

5. Slutsker LM, Khoromana CO, Payne D, et al. Mefloquine therapy for Plasmodium falciparum. malaria in children under 5 years of age in Malawi: in vivo/in vitro efficacy and correlation of drug concentration with parasitological outcome. Bull World Health Organ 1990;68:53-59.

6. Lobel HO, Miani M, Eng T, et al. Long-term malaria prophylaxis with weekly mefloquine. Lancet 1993;341:848-51.

7. Schlagenhauf P. Mefloquine for malaria chemoprophylaxis 1992-1998: a review. J Travel Med 1999;6:122-133.

8. International Travel and Health 2005. Chapter 7: Malaria. Geneva: World Health Organization; 2005. Available at: whqlibdoc.who.int/publications/2005/9241580364_chap7.pdf. Accessed on October 11, 2007.

Issue
The Journal of Family Practice - 56(11)
Issue
The Journal of Family Practice - 56(11)
Page Number
950-951
Page Number
950-951
Publications
Publications
Topics
Article Type
Display Headline
When should travelers begin malaria prophylaxis?
Display Headline
When should travelers begin malaria prophylaxis?
Legacy Keywords
Steven L. Clark;MD; Paul Crawford;MD; William Nichols;MLS; Brian V. Reamy ;MD; malaria; prophylaxis; anopheles; chloroquine; resistant; Plasmodium falciparum; travel; atovaquone; proguanil; doxycycline; primaquine; mefloquine; schedule; infections; parasite; CDC; WHO
Legacy Keywords
Steven L. Clark;MD; Paul Crawford;MD; William Nichols;MLS; Brian V. Reamy ;MD; malaria; prophylaxis; anopheles; chloroquine; resistant; Plasmodium falciparum; travel; atovaquone; proguanil; doxycycline; primaquine; mefloquine; schedule; infections; parasite; CDC; WHO
Sections
PURLs Copyright

Evidence-based answers from the Family Physicians Inquiries Network

Disallow All Ads
Article PDF Media

Which nondrug alternatives can help with insomnia?

Article Type
Changed
Mon, 01/14/2019 - 11:21
Display Headline
Which nondrug alternatives can help with insomnia?
EVIDENCE-BASED ANSWER

Cognitive behavioral therapy (CBT) interventions—particularly stimulus control and sleep hygiene—are well-validated, effective treatments for chronic insomnia that are equivalent or superior to pharmacological interventions (strength of recommendation: A, based on systematic reviews). The long-term efficacy of CBT interventions, and their successful implementation by primary care physicians (as compared with behavioral science providers), is unclear.

Clinical commentary

Can I provide these interventions without a referral?
John D Hallgren, Lt Col, USAF, MC
Uniformed Services University of the Health Sciences, RAF Menwith Hill, UK

A large proportion of people in my patient population are shift workers, so chronic insomnia plays a large role in my daily workload, both directly and indirectly. This summary tells me that I have a proven and equally efficacious alternative to drugs for these sufferers—which is great.

However, I was disappointed to see that none of the CBT interventions were performed by family physicians in the office. So the good news is that I have a nondrug intervention for insomnia; the bad news is I don’t know if it’s something I can provide without a referral. Maybe it’s time for some practice-based research to see if that is possible.

Evidence summary

Approximately 10% to 15% of adults complain of chronic insomnia, best defined as difficulty initiating or maintaining sleep 3 or more nights per week for 6 months or longer, with secondary impairments in daytime functioning, including fatigue and disturbed mood.1-3

Behavioral and psychological treatments have emerged as increasingly popular adjunctive interventions to pharmacotherapy and as independent interventions for chronic insomnia. No evidence exists that behavioral treatments have adverse effects.1

Sleep hygiene, relaxation training, and cognitive therapy improve sleep

CBT interventions are based on the notion that distorted thoughts about sleep and learned behavior patterns hyperarouse the central nervous system and deregulate sleep cycles, resulting in chronic insomnia.4 CBT interventions combine empirically tested behavioral, cognitive, and educational procedures to alter faulty beliefs and attitudes, modify sleep habits, and regulate sleep-wake schedules.3

These interventions include stimulus control, sleep hygiene, sleep restriction, relaxation training, and cognitive therapy.5 These methods can be used separately; however, they are increasingly being used together to treat the complexities of individual patients.5

 

Five recent high-quality randomized control trials (RCTs) confirmed findings from earlier RCTs that CBT methods improve sleep.5 Compared with those given a placebo or placed on a waiting list, CBT-treated patients in these RCTs reported clinically significant improvements in sleep onset latency, sleep efficiency, time awake after sleep onset, and total sleep time. In one RCT, 64% of CBT patients had improvements in sleep efficiency and time awake after sleep onset, compared with 8% who improved with a placebo intervention (number needed to treat [NNT]=1.8).5 Further, sleep onset latency for primary care patients with chronic insomnia was decreased from 61 to 28 minutes, compared with 74 to 70 minutes for a waiting-list group.5 The maintenance of sleep gains from CBT beyond 1 year is unknown since no published RCT clinical trials to date have lasted longer than 12 months.1

 

 

An important related meta-analysis of 21 studies validated behavior therapy, and revealed CBT reduced sleep onset latency by an additional 8.8 minutes over medication (95% confidence interval, 0.17–1.04 minutes).6 Although not superior on other outcomes, behavior therapy produced similar short-term results to pharmacotherapy across all other sleep measures, without attendant medication side effects.

Stimulus control is the most effective CBT intervention

 

A recent systematic review with meta-analysis of 37 clinical investigations determined that stimulus control was the most effective CBT intervention.3 Stimulus control consists of 5 basic instructions (TABLE) designed to help the patient reassociate sleep stimuli (ie, bed/bedroom) with falling asleep and establishing consistent sleep-wake schedules. These 5 instructions are frequently used in combination with CBT sleep hygiene techniques (TABLE) and can be easily integrated into the office setting.3,4

Among the CBT techniques, stimulus control and sleep hygiene are the least time-consuming and may be more easily applied in the primary care setting; however, minimal research has been done into the specific incorporation of CBT into primary care settings.

Researchers conducting a single-blind randomized group study in a Veterans Affairs primary care clinic concluded that an abbreviated CBT approach with two 25-minute sessions effectively improved participant sleep onset latency, and time awake after sleep onset.7 Researchers reviewed participants’ sleep logs and a behavioral health provider offered patients a condensed education on sleep hygiene, stimulus control, and sleep restrictions strategies. The study was limited because of small sample size (<25). Generalizability to practice is restricted because sessions were conducted by a behavioral health provider, not a family physician.

TABLE
Patient needs a good night’s sleep? Offer this advice

STIMULUS CONTROL INSTRUCTIONS3
  • Don’t go to bed until you are sleepy
  • Use the bed/bedroom only for sleeping (don’t read, watch TV, eat, or worry)
  • Get out of bed when unable to sleep after 15 minutes; do something relaxing and avoid stimulating activity/thoughts
  • Arise from bed at the same time every day
  • Do not nap during the day
SLEEP HYGIENE INSTRUCTIONS4
  • Sleep only as much as you need to feel refreshed during the following day
  • Exercise regularly
  • Make sure your bedroom is comfortable and free from disturbing light and noise
  • Make sure your bedroom is at a comfortable temperature during the night
  • Eat regular meals and do not go to bed hungry
  • Avoid drinking too many fluids in the evening
  • Reduce your caffeine intake
  • Avoid drinking alcohol—especially in the evening
  • Avoid smoking at night when you are having trouble sleeping
  • Don’t try too hard to fall asleep
  • Put the clock under the bed or turn it so you can’t see it

Recommendations from others

The Agency for Healthcare Research and Quality recommends CBT as an effective treatment in the management of chronic .8 It also recommends that further large-scale RCTs be conducted to establish CBT’s effectiveness across subsets of the population of individuals with chronic (ie, gender, age, shift workers, and those with psychiatric illnesses).

The American Psychological Association (APA) recommends CBT as the “treatment of choice” for chronic , with 70% to 80% of patients showing a treatment response.9

Acknowledgments

The opinions and assertions contained herein are the private views of the authors and not to be construed as official, or as reflecting the views of the US Air Force Medical Service or the US Air Force at large.

References

1. NIH State-of-the-Science Conference Statement on Manifestations and Management of Chronic Insomnia in Adults. NIH Consensus Science Statement. 2005; 22(2). Available at: consensus.nih.gov/2005/2005InsomniaSOS026main.htm. Accessed on September 4, 2007.

2. Ohayon M. Epidemiology of insomnia: What we know and what we still need to learn. Sleep Med Rev 2002;6:97-111.

3. Morin CM. Cognitive-behavioral approaches to the treatment of insomnia. J Clin Psychiatry 2004;65 Suppl 16:33-40.

4. Smith MT, Neubauer DN. Cognitive behavioral therapy for chronic insomnia. Clinical Cornerstone 2003;5:1-9.

5. Morin CM, Bootzin RR, Buysse DJ, Edinger JD, Espie CA. Psychological and behavioral treatment of insomnia: Update of the recent evidence (1998–2004). Sleep 2006;29:1398-1413.

6. Smith MT, Perlis ML, Park A, et al. Comparative meta-analysis of pharmacotherapy and behavior therapy for persistent insomnia. Am J Psychiatry 2002;159:5-11.

7. Edinger JD, Sampson WS. A primary care “friendly” cognitive behavioral insomnia therapy. Sleep 2003;26:177-182.

8. Buscemi N, Vandermeer B, Friesen C, et al. Manifestations of chronic insomnia in adults. Evidence report/technology assessment No. 125. (Prepared by the University of Alberta Evidence-based Practice Center, under Contract N. C400000021.) AHRQ Publication No. 05-E021-1. Rockville, Md: Agency for Healthcare Research and Quality. June 2005. Available at: www.ahrq.gov/downloads/pub/evidence/pdf/insomnia/insomnia.pdf. Accessed on September 4, 2007.

9. American Psychological Association Web site. Getting a good night’s sleep with the help of psychology. Available at: www.psychologymatters.org/insomnia.html. Accessed on September 4, 2007.

Article PDF
Author and Disclosure Information

James D. Whitworth, PhD
Brian K. Crownover, MD, FAAFP
HQ Air Armament Center, Family Medicine Residency, Eglin Air Force Base, Fla

William Nichols, MLS
Eglin Air Force Base, 96th Medical Group, Eglin Air Force Base, Fla

Issue
The Journal of Family Practice - 56(10)
Publications
Topics
Page Number
836-840
Legacy Keywords
insomnia; nondrug; treatment; sleep; disorder; CBT; cognitive; behavioral; therapy; sleep hygiene; stimulus control; James D. Whitworth PhD; Brian K. Crownover MD; William Nichols MLS; Lt Col John D. Hallgren
Sections
Author and Disclosure Information

James D. Whitworth, PhD
Brian K. Crownover, MD, FAAFP
HQ Air Armament Center, Family Medicine Residency, Eglin Air Force Base, Fla

William Nichols, MLS
Eglin Air Force Base, 96th Medical Group, Eglin Air Force Base, Fla

Author and Disclosure Information

James D. Whitworth, PhD
Brian K. Crownover, MD, FAAFP
HQ Air Armament Center, Family Medicine Residency, Eglin Air Force Base, Fla

William Nichols, MLS
Eglin Air Force Base, 96th Medical Group, Eglin Air Force Base, Fla

Article PDF
Article PDF
EVIDENCE-BASED ANSWER

Cognitive behavioral therapy (CBT) interventions—particularly stimulus control and sleep hygiene—are well-validated, effective treatments for chronic insomnia that are equivalent or superior to pharmacological interventions (strength of recommendation: A, based on systematic reviews). The long-term efficacy of CBT interventions, and their successful implementation by primary care physicians (as compared with behavioral science providers), is unclear.

Clinical commentary

Can I provide these interventions without a referral?
John D Hallgren, Lt Col, USAF, MC
Uniformed Services University of the Health Sciences, RAF Menwith Hill, UK

A large proportion of people in my patient population are shift workers, so chronic insomnia plays a large role in my daily workload, both directly and indirectly. This summary tells me that I have a proven and equally efficacious alternative to drugs for these sufferers—which is great.

However, I was disappointed to see that none of the CBT interventions were performed by family physicians in the office. So the good news is that I have a nondrug intervention for insomnia; the bad news is I don’t know if it’s something I can provide without a referral. Maybe it’s time for some practice-based research to see if that is possible.

Evidence summary

Approximately 10% to 15% of adults complain of chronic insomnia, best defined as difficulty initiating or maintaining sleep 3 or more nights per week for 6 months or longer, with secondary impairments in daytime functioning, including fatigue and disturbed mood.1-3

Behavioral and psychological treatments have emerged as increasingly popular adjunctive interventions to pharmacotherapy and as independent interventions for chronic insomnia. No evidence exists that behavioral treatments have adverse effects.1

Sleep hygiene, relaxation training, and cognitive therapy improve sleep

CBT interventions are based on the notion that distorted thoughts about sleep and learned behavior patterns hyperarouse the central nervous system and deregulate sleep cycles, resulting in chronic insomnia.4 CBT interventions combine empirically tested behavioral, cognitive, and educational procedures to alter faulty beliefs and attitudes, modify sleep habits, and regulate sleep-wake schedules.3

These interventions include stimulus control, sleep hygiene, sleep restriction, relaxation training, and cognitive therapy.5 These methods can be used separately; however, they are increasingly being used together to treat the complexities of individual patients.5

 

Five recent high-quality randomized control trials (RCTs) confirmed findings from earlier RCTs that CBT methods improve sleep.5 Compared with those given a placebo or placed on a waiting list, CBT-treated patients in these RCTs reported clinically significant improvements in sleep onset latency, sleep efficiency, time awake after sleep onset, and total sleep time. In one RCT, 64% of CBT patients had improvements in sleep efficiency and time awake after sleep onset, compared with 8% who improved with a placebo intervention (number needed to treat [NNT]=1.8).5 Further, sleep onset latency for primary care patients with chronic insomnia was decreased from 61 to 28 minutes, compared with 74 to 70 minutes for a waiting-list group.5 The maintenance of sleep gains from CBT beyond 1 year is unknown since no published RCT clinical trials to date have lasted longer than 12 months.1

 

 

An important related meta-analysis of 21 studies validated behavior therapy, and revealed CBT reduced sleep onset latency by an additional 8.8 minutes over medication (95% confidence interval, 0.17–1.04 minutes).6 Although not superior on other outcomes, behavior therapy produced similar short-term results to pharmacotherapy across all other sleep measures, without attendant medication side effects.

Stimulus control is the most effective CBT intervention

 

A recent systematic review with meta-analysis of 37 clinical investigations determined that stimulus control was the most effective CBT intervention.3 Stimulus control consists of 5 basic instructions (TABLE) designed to help the patient reassociate sleep stimuli (ie, bed/bedroom) with falling asleep and establishing consistent sleep-wake schedules. These 5 instructions are frequently used in combination with CBT sleep hygiene techniques (TABLE) and can be easily integrated into the office setting.3,4

Among the CBT techniques, stimulus control and sleep hygiene are the least time-consuming and may be more easily applied in the primary care setting; however, minimal research has been done into the specific incorporation of CBT into primary care settings.

Researchers conducting a single-blind randomized group study in a Veterans Affairs primary care clinic concluded that an abbreviated CBT approach with two 25-minute sessions effectively improved participant sleep onset latency, and time awake after sleep onset.7 Researchers reviewed participants’ sleep logs and a behavioral health provider offered patients a condensed education on sleep hygiene, stimulus control, and sleep restrictions strategies. The study was limited because of small sample size (<25). Generalizability to practice is restricted because sessions were conducted by a behavioral health provider, not a family physician.

TABLE
Patient needs a good night’s sleep? Offer this advice

STIMULUS CONTROL INSTRUCTIONS3
  • Don’t go to bed until you are sleepy
  • Use the bed/bedroom only for sleeping (don’t read, watch TV, eat, or worry)
  • Get out of bed when unable to sleep after 15 minutes; do something relaxing and avoid stimulating activity/thoughts
  • Arise from bed at the same time every day
  • Do not nap during the day
SLEEP HYGIENE INSTRUCTIONS4
  • Sleep only as much as you need to feel refreshed during the following day
  • Exercise regularly
  • Make sure your bedroom is comfortable and free from disturbing light and noise
  • Make sure your bedroom is at a comfortable temperature during the night
  • Eat regular meals and do not go to bed hungry
  • Avoid drinking too many fluids in the evening
  • Reduce your caffeine intake
  • Avoid drinking alcohol—especially in the evening
  • Avoid smoking at night when you are having trouble sleeping
  • Don’t try too hard to fall asleep
  • Put the clock under the bed or turn it so you can’t see it

Recommendations from others

The Agency for Healthcare Research and Quality recommends CBT as an effective treatment in the management of chronic .8 It also recommends that further large-scale RCTs be conducted to establish CBT’s effectiveness across subsets of the population of individuals with chronic (ie, gender, age, shift workers, and those with psychiatric illnesses).

The American Psychological Association (APA) recommends CBT as the “treatment of choice” for chronic , with 70% to 80% of patients showing a treatment response.9

Acknowledgments

The opinions and assertions contained herein are the private views of the authors and not to be construed as official, or as reflecting the views of the US Air Force Medical Service or the US Air Force at large.

EVIDENCE-BASED ANSWER

Cognitive behavioral therapy (CBT) interventions—particularly stimulus control and sleep hygiene—are well-validated, effective treatments for chronic insomnia that are equivalent or superior to pharmacological interventions (strength of recommendation: A, based on systematic reviews). The long-term efficacy of CBT interventions, and their successful implementation by primary care physicians (as compared with behavioral science providers), is unclear.

Clinical commentary

Can I provide these interventions without a referral?
John D Hallgren, Lt Col, USAF, MC
Uniformed Services University of the Health Sciences, RAF Menwith Hill, UK

A large proportion of people in my patient population are shift workers, so chronic insomnia plays a large role in my daily workload, both directly and indirectly. This summary tells me that I have a proven and equally efficacious alternative to drugs for these sufferers—which is great.

However, I was disappointed to see that none of the CBT interventions were performed by family physicians in the office. So the good news is that I have a nondrug intervention for insomnia; the bad news is I don’t know if it’s something I can provide without a referral. Maybe it’s time for some practice-based research to see if that is possible.

Evidence summary

Approximately 10% to 15% of adults complain of chronic insomnia, best defined as difficulty initiating or maintaining sleep 3 or more nights per week for 6 months or longer, with secondary impairments in daytime functioning, including fatigue and disturbed mood.1-3

Behavioral and psychological treatments have emerged as increasingly popular adjunctive interventions to pharmacotherapy and as independent interventions for chronic insomnia. No evidence exists that behavioral treatments have adverse effects.1

Sleep hygiene, relaxation training, and cognitive therapy improve sleep

CBT interventions are based on the notion that distorted thoughts about sleep and learned behavior patterns hyperarouse the central nervous system and deregulate sleep cycles, resulting in chronic insomnia.4 CBT interventions combine empirically tested behavioral, cognitive, and educational procedures to alter faulty beliefs and attitudes, modify sleep habits, and regulate sleep-wake schedules.3

These interventions include stimulus control, sleep hygiene, sleep restriction, relaxation training, and cognitive therapy.5 These methods can be used separately; however, they are increasingly being used together to treat the complexities of individual patients.5

 

Five recent high-quality randomized control trials (RCTs) confirmed findings from earlier RCTs that CBT methods improve sleep.5 Compared with those given a placebo or placed on a waiting list, CBT-treated patients in these RCTs reported clinically significant improvements in sleep onset latency, sleep efficiency, time awake after sleep onset, and total sleep time. In one RCT, 64% of CBT patients had improvements in sleep efficiency and time awake after sleep onset, compared with 8% who improved with a placebo intervention (number needed to treat [NNT]=1.8).5 Further, sleep onset latency for primary care patients with chronic insomnia was decreased from 61 to 28 minutes, compared with 74 to 70 minutes for a waiting-list group.5 The maintenance of sleep gains from CBT beyond 1 year is unknown since no published RCT clinical trials to date have lasted longer than 12 months.1

 

 

An important related meta-analysis of 21 studies validated behavior therapy, and revealed CBT reduced sleep onset latency by an additional 8.8 minutes over medication (95% confidence interval, 0.17–1.04 minutes).6 Although not superior on other outcomes, behavior therapy produced similar short-term results to pharmacotherapy across all other sleep measures, without attendant medication side effects.

Stimulus control is the most effective CBT intervention

 

A recent systematic review with meta-analysis of 37 clinical investigations determined that stimulus control was the most effective CBT intervention.3 Stimulus control consists of 5 basic instructions (TABLE) designed to help the patient reassociate sleep stimuli (ie, bed/bedroom) with falling asleep and establishing consistent sleep-wake schedules. These 5 instructions are frequently used in combination with CBT sleep hygiene techniques (TABLE) and can be easily integrated into the office setting.3,4

Among the CBT techniques, stimulus control and sleep hygiene are the least time-consuming and may be more easily applied in the primary care setting; however, minimal research has been done into the specific incorporation of CBT into primary care settings.

Researchers conducting a single-blind randomized group study in a Veterans Affairs primary care clinic concluded that an abbreviated CBT approach with two 25-minute sessions effectively improved participant sleep onset latency, and time awake after sleep onset.7 Researchers reviewed participants’ sleep logs and a behavioral health provider offered patients a condensed education on sleep hygiene, stimulus control, and sleep restrictions strategies. The study was limited because of small sample size (<25). Generalizability to practice is restricted because sessions were conducted by a behavioral health provider, not a family physician.

TABLE
Patient needs a good night’s sleep? Offer this advice

STIMULUS CONTROL INSTRUCTIONS3
  • Don’t go to bed until you are sleepy
  • Use the bed/bedroom only for sleeping (don’t read, watch TV, eat, or worry)
  • Get out of bed when unable to sleep after 15 minutes; do something relaxing and avoid stimulating activity/thoughts
  • Arise from bed at the same time every day
  • Do not nap during the day
SLEEP HYGIENE INSTRUCTIONS4
  • Sleep only as much as you need to feel refreshed during the following day
  • Exercise regularly
  • Make sure your bedroom is comfortable and free from disturbing light and noise
  • Make sure your bedroom is at a comfortable temperature during the night
  • Eat regular meals and do not go to bed hungry
  • Avoid drinking too many fluids in the evening
  • Reduce your caffeine intake
  • Avoid drinking alcohol—especially in the evening
  • Avoid smoking at night when you are having trouble sleeping
  • Don’t try too hard to fall asleep
  • Put the clock under the bed or turn it so you can’t see it

Recommendations from others

The Agency for Healthcare Research and Quality recommends CBT as an effective treatment in the management of chronic .8 It also recommends that further large-scale RCTs be conducted to establish CBT’s effectiveness across subsets of the population of individuals with chronic (ie, gender, age, shift workers, and those with psychiatric illnesses).

The American Psychological Association (APA) recommends CBT as the “treatment of choice” for chronic , with 70% to 80% of patients showing a treatment response.9

Acknowledgments

The opinions and assertions contained herein are the private views of the authors and not to be construed as official, or as reflecting the views of the US Air Force Medical Service or the US Air Force at large.

References

1. NIH State-of-the-Science Conference Statement on Manifestations and Management of Chronic Insomnia in Adults. NIH Consensus Science Statement. 2005; 22(2). Available at: consensus.nih.gov/2005/2005InsomniaSOS026main.htm. Accessed on September 4, 2007.

2. Ohayon M. Epidemiology of insomnia: What we know and what we still need to learn. Sleep Med Rev 2002;6:97-111.

3. Morin CM. Cognitive-behavioral approaches to the treatment of insomnia. J Clin Psychiatry 2004;65 Suppl 16:33-40.

4. Smith MT, Neubauer DN. Cognitive behavioral therapy for chronic insomnia. Clinical Cornerstone 2003;5:1-9.

5. Morin CM, Bootzin RR, Buysse DJ, Edinger JD, Espie CA. Psychological and behavioral treatment of insomnia: Update of the recent evidence (1998–2004). Sleep 2006;29:1398-1413.

6. Smith MT, Perlis ML, Park A, et al. Comparative meta-analysis of pharmacotherapy and behavior therapy for persistent insomnia. Am J Psychiatry 2002;159:5-11.

7. Edinger JD, Sampson WS. A primary care “friendly” cognitive behavioral insomnia therapy. Sleep 2003;26:177-182.

8. Buscemi N, Vandermeer B, Friesen C, et al. Manifestations of chronic insomnia in adults. Evidence report/technology assessment No. 125. (Prepared by the University of Alberta Evidence-based Practice Center, under Contract N. C400000021.) AHRQ Publication No. 05-E021-1. Rockville, Md: Agency for Healthcare Research and Quality. June 2005. Available at: www.ahrq.gov/downloads/pub/evidence/pdf/insomnia/insomnia.pdf. Accessed on September 4, 2007.

9. American Psychological Association Web site. Getting a good night’s sleep with the help of psychology. Available at: www.psychologymatters.org/insomnia.html. Accessed on September 4, 2007.

References

1. NIH State-of-the-Science Conference Statement on Manifestations and Management of Chronic Insomnia in Adults. NIH Consensus Science Statement. 2005; 22(2). Available at: consensus.nih.gov/2005/2005InsomniaSOS026main.htm. Accessed on September 4, 2007.

2. Ohayon M. Epidemiology of insomnia: What we know and what we still need to learn. Sleep Med Rev 2002;6:97-111.

3. Morin CM. Cognitive-behavioral approaches to the treatment of insomnia. J Clin Psychiatry 2004;65 Suppl 16:33-40.

4. Smith MT, Neubauer DN. Cognitive behavioral therapy for chronic insomnia. Clinical Cornerstone 2003;5:1-9.

5. Morin CM, Bootzin RR, Buysse DJ, Edinger JD, Espie CA. Psychological and behavioral treatment of insomnia: Update of the recent evidence (1998–2004). Sleep 2006;29:1398-1413.

6. Smith MT, Perlis ML, Park A, et al. Comparative meta-analysis of pharmacotherapy and behavior therapy for persistent insomnia. Am J Psychiatry 2002;159:5-11.

7. Edinger JD, Sampson WS. A primary care “friendly” cognitive behavioral insomnia therapy. Sleep 2003;26:177-182.

8. Buscemi N, Vandermeer B, Friesen C, et al. Manifestations of chronic insomnia in adults. Evidence report/technology assessment No. 125. (Prepared by the University of Alberta Evidence-based Practice Center, under Contract N. C400000021.) AHRQ Publication No. 05-E021-1. Rockville, Md: Agency for Healthcare Research and Quality. June 2005. Available at: www.ahrq.gov/downloads/pub/evidence/pdf/insomnia/insomnia.pdf. Accessed on September 4, 2007.

9. American Psychological Association Web site. Getting a good night’s sleep with the help of psychology. Available at: www.psychologymatters.org/insomnia.html. Accessed on September 4, 2007.

Issue
The Journal of Family Practice - 56(10)
Issue
The Journal of Family Practice - 56(10)
Page Number
836-840
Page Number
836-840
Publications
Publications
Topics
Article Type
Display Headline
Which nondrug alternatives can help with insomnia?
Display Headline
Which nondrug alternatives can help with insomnia?
Legacy Keywords
insomnia; nondrug; treatment; sleep; disorder; CBT; cognitive; behavioral; therapy; sleep hygiene; stimulus control; James D. Whitworth PhD; Brian K. Crownover MD; William Nichols MLS; Lt Col John D. Hallgren
Legacy Keywords
insomnia; nondrug; treatment; sleep; disorder; CBT; cognitive; behavioral; therapy; sleep hygiene; stimulus control; James D. Whitworth PhD; Brian K. Crownover MD; William Nichols MLS; Lt Col John D. Hallgren
Sections
PURLs Copyright

Evidence-based answers from the Family Physicians Inquiries Network

Disallow All Ads
Alternative CME
Article PDF Media

Should we use appetite stimulants for malnourished elderly patients?

Article Type
Changed
Mon, 01/14/2019 - 11:20
Display Headline
Should we use appetite stimulants for malnourished elderly patients?
EVIDENCE-BASED ANSWER

Probably not. Only 1 appetite stimulate, megestrol acetate oral suspension (Megace) at 400 mg or 800 mg daily, has been studied in this population. The data show only limited benefit, mixed outcomes, and potential harm (strength of recommendation: B, based on small, randomized, controlled trials).

Clinical commentary

Good advice for a common problem
Kayleen P. Papin, MD
Medical College of Wisconsin, Milwaukee

This question hits home for me. I recently sat down with the husband, and main caregiver, of a woman with advanced dementia. The woman eats very little and is losing weight despite her husband’s great efforts at encouraging her to eat. Under the care of another physician, she had been given megestrol acetate and there had been some improvement. Her visit to my office was an opportunity to continue an ongoing conversation with her husband about his wife’s overall decline, her advancing dementia, and the sorrow he was feeling over her failing health.

Should we use appetite stimulants in malnourished elderly patients? “probably not.” that is a good place to start to avoid harm to our most frail, declining, elderly patients for whom we care. That leaves open flexibility to patient, family, and caregiver preferences, but reminds us that the most important part of caring for these patients and their families is clear, compassionate communication regarding goals and expectations.

 

Evidence summary

Although a number of studies have evaluated various appetite stimulants—megestrol, dronabinol (Marinol), cyproheptadine (Periactin), thalidomide (Thalomid), pentoxifylline (Pentoxil/Trental), nandrolone decanoate (DecaDurabolin), oxandrolone (Oxandrin), and corticosteroids—in patients with AIDS, anorexia cachexia syndrome, and advanced cancer, only megestrol has been studied in malnourished elderly patients.

Two studies, mixed results

One placebo-controlled randomized clinical trial studied 45 malnourished patients who were recently discharged from an acute care hospital to a nursing home. The patients (predominately female, with a mean age of 83) were randomized into 4 treatment arms (placebo or megestrol 200 mg, 400 mg, or 800 mg daily) and followed for 63 days.

Only those receiving megestrol (400 mg or 800 mg daily) demonstrated a statistically significant increase in patient appetite and a dose-responsive increase in prealbumin level at the 20 day interim analysis (7.5 and 9.0 mg/dL, respectfully). But at the final assessment (63 days), only the 400-mg dose maintained a statistically significant increase in prealbumin over placebo. However, there was no significant improvement in serum albumin or clinical endpoints (weight, functional status, or health-related quality of life).1

In contrast, an earlier Veterans Administration (and predominantly male) study showed 13/21 of those treated with megestrol (800 mg daily for 12 weeks) noted weight gain (≥4 lb sustained at 3 months post-treatment), compared with 5/23 of those receiving placebo (number needed to treat [NNT]=2.5).2 Of note, only 9/26 patients had sustained weight gain in the megestrol group at the 12-month endpoint post-treatment, comparable with 7/25 in the placebo group.

Some small, but statistically significant, score improvements were noted during the treatment period in appetite and enjoyment of life; however, no differences were noted in scores on the more widely accepted Geriatric Depression Scale.

Adverse effects

As in all therapeutic interventions, benefit must be balanced against risk. The Megace ES package insert notes the following potential adverse effects: diarrhea, cardiomyopathy, palpitation, hepatomegaly, leukopenia, edema, paresthesia, confusion, convulsion, depression, neuropathy, hypesthesia and abnormal thinking, thrombophlebitis, pulmonary embolism, and glucose intolerance.3

To date, the prevalence rates of these potential adverse effects have only been studied in patients with AIDS. No data reflecting potential rates in elderly patients have been published.

Recommendations from others

The American Geriatric Society4 made 3 comments on appetite stimulation:

  1. There are no FDA-approved drugs available for the promotion of weight gain in older adults.
  2. A minority of patients receiving mirtazapine report appetite stimulation and weight gain.
  3. All drugs used for appetite have substantial potential adverse events.

We found only 1 national guideline on this topic: Unintentional Weight Loss in the Elderly from the University of Texas School of Nursing.5 The guideline indicates that drugs should not be used as first-line intervention in the elderly, as there has been inadequate testing in this population. Benefits are restricted to small weight gains without indication of decreased morbidity or mortality, improved quality of life, or improved functional ability.

Acknowledgments

The opinions and assertions contained herein are the private views of the author and not to be construed as official, or as reflecting the views of the US Air Force Medical Service or the US Air Force at large.

References

1. Reuben DB, Hirsch SH, Zhou K, Greendale GA. The effects of megestrol acetate suspension for elderly patients with reduced appetite after hospitalization: a phase II randomized clinical trial. J Am Geriatr Soc 2005;53:970-975.

2. Yeh SS, Wu SY, Lee TP, et al. Improvement in quality-of-life measures and stimulation of weight gain after treatment with megestrol acetate or suspension in geriatric cachexia: results of a double-blind, placebo controlled study. J Am Geriatr Soc 2001;48:485-492.

3. Megace Physicians’ Desk Reference 61st ed. Montvale, NJ: Thomson; 2007:2461-2463.

4. Malnutrition. Geriatrics at Your Fingertips [website]. Available at: www.geriatricsatyourfingertips.org/ebook/gayf_20.asp. Accessed August 6, 2007.

5. University of Texas, School of Nursing. Unintentional Weight Loss in the Elderly. Austin, Tex: University of Texas, School of Nursing; 2006. Available at: www.guideline.gov/summary/summary.aspx?doc_id=9435. Accessed August 6, 2007.

Article PDF
Author and Disclosure Information

Robert K. Persons, DO, FAAP
William Nichols, MLS
HQ Air Armament Center (Eglin) Family Medicine Residency, Eglin Air Force Base, Fla

Issue
The Journal of Family Practice - 56(9)
Publications
Topics
Page Number
761-762
Legacy Keywords
Megace; appetite; stimulant; weight; loss; unintended; elderly; aged; malnourished; Robert K. Persons DO; William Nichols MLS; Kayleen P. Papin MD
Sections
Author and Disclosure Information

Robert K. Persons, DO, FAAP
William Nichols, MLS
HQ Air Armament Center (Eglin) Family Medicine Residency, Eglin Air Force Base, Fla

Author and Disclosure Information

Robert K. Persons, DO, FAAP
William Nichols, MLS
HQ Air Armament Center (Eglin) Family Medicine Residency, Eglin Air Force Base, Fla

Article PDF
Article PDF
EVIDENCE-BASED ANSWER

Probably not. Only 1 appetite stimulate, megestrol acetate oral suspension (Megace) at 400 mg or 800 mg daily, has been studied in this population. The data show only limited benefit, mixed outcomes, and potential harm (strength of recommendation: B, based on small, randomized, controlled trials).

Clinical commentary

Good advice for a common problem
Kayleen P. Papin, MD
Medical College of Wisconsin, Milwaukee

This question hits home for me. I recently sat down with the husband, and main caregiver, of a woman with advanced dementia. The woman eats very little and is losing weight despite her husband’s great efforts at encouraging her to eat. Under the care of another physician, she had been given megestrol acetate and there had been some improvement. Her visit to my office was an opportunity to continue an ongoing conversation with her husband about his wife’s overall decline, her advancing dementia, and the sorrow he was feeling over her failing health.

Should we use appetite stimulants in malnourished elderly patients? “probably not.” that is a good place to start to avoid harm to our most frail, declining, elderly patients for whom we care. That leaves open flexibility to patient, family, and caregiver preferences, but reminds us that the most important part of caring for these patients and their families is clear, compassionate communication regarding goals and expectations.

 

Evidence summary

Although a number of studies have evaluated various appetite stimulants—megestrol, dronabinol (Marinol), cyproheptadine (Periactin), thalidomide (Thalomid), pentoxifylline (Pentoxil/Trental), nandrolone decanoate (DecaDurabolin), oxandrolone (Oxandrin), and corticosteroids—in patients with AIDS, anorexia cachexia syndrome, and advanced cancer, only megestrol has been studied in malnourished elderly patients.

Two studies, mixed results

One placebo-controlled randomized clinical trial studied 45 malnourished patients who were recently discharged from an acute care hospital to a nursing home. The patients (predominately female, with a mean age of 83) were randomized into 4 treatment arms (placebo or megestrol 200 mg, 400 mg, or 800 mg daily) and followed for 63 days.

Only those receiving megestrol (400 mg or 800 mg daily) demonstrated a statistically significant increase in patient appetite and a dose-responsive increase in prealbumin level at the 20 day interim analysis (7.5 and 9.0 mg/dL, respectfully). But at the final assessment (63 days), only the 400-mg dose maintained a statistically significant increase in prealbumin over placebo. However, there was no significant improvement in serum albumin or clinical endpoints (weight, functional status, or health-related quality of life).1

In contrast, an earlier Veterans Administration (and predominantly male) study showed 13/21 of those treated with megestrol (800 mg daily for 12 weeks) noted weight gain (≥4 lb sustained at 3 months post-treatment), compared with 5/23 of those receiving placebo (number needed to treat [NNT]=2.5).2 Of note, only 9/26 patients had sustained weight gain in the megestrol group at the 12-month endpoint post-treatment, comparable with 7/25 in the placebo group.

Some small, but statistically significant, score improvements were noted during the treatment period in appetite and enjoyment of life; however, no differences were noted in scores on the more widely accepted Geriatric Depression Scale.

Adverse effects

As in all therapeutic interventions, benefit must be balanced against risk. The Megace ES package insert notes the following potential adverse effects: diarrhea, cardiomyopathy, palpitation, hepatomegaly, leukopenia, edema, paresthesia, confusion, convulsion, depression, neuropathy, hypesthesia and abnormal thinking, thrombophlebitis, pulmonary embolism, and glucose intolerance.3

To date, the prevalence rates of these potential adverse effects have only been studied in patients with AIDS. No data reflecting potential rates in elderly patients have been published.

Recommendations from others

The American Geriatric Society4 made 3 comments on appetite stimulation:

  1. There are no FDA-approved drugs available for the promotion of weight gain in older adults.
  2. A minority of patients receiving mirtazapine report appetite stimulation and weight gain.
  3. All drugs used for appetite have substantial potential adverse events.

We found only 1 national guideline on this topic: Unintentional Weight Loss in the Elderly from the University of Texas School of Nursing.5 The guideline indicates that drugs should not be used as first-line intervention in the elderly, as there has been inadequate testing in this population. Benefits are restricted to small weight gains without indication of decreased morbidity or mortality, improved quality of life, or improved functional ability.

Acknowledgments

The opinions and assertions contained herein are the private views of the author and not to be construed as official, or as reflecting the views of the US Air Force Medical Service or the US Air Force at large.

EVIDENCE-BASED ANSWER

Probably not. Only 1 appetite stimulate, megestrol acetate oral suspension (Megace) at 400 mg or 800 mg daily, has been studied in this population. The data show only limited benefit, mixed outcomes, and potential harm (strength of recommendation: B, based on small, randomized, controlled trials).

Clinical commentary

Good advice for a common problem
Kayleen P. Papin, MD
Medical College of Wisconsin, Milwaukee

This question hits home for me. I recently sat down with the husband, and main caregiver, of a woman with advanced dementia. The woman eats very little and is losing weight despite her husband’s great efforts at encouraging her to eat. Under the care of another physician, she had been given megestrol acetate and there had been some improvement. Her visit to my office was an opportunity to continue an ongoing conversation with her husband about his wife’s overall decline, her advancing dementia, and the sorrow he was feeling over her failing health.

Should we use appetite stimulants in malnourished elderly patients? “probably not.” that is a good place to start to avoid harm to our most frail, declining, elderly patients for whom we care. That leaves open flexibility to patient, family, and caregiver preferences, but reminds us that the most important part of caring for these patients and their families is clear, compassionate communication regarding goals and expectations.

 

Evidence summary

Although a number of studies have evaluated various appetite stimulants—megestrol, dronabinol (Marinol), cyproheptadine (Periactin), thalidomide (Thalomid), pentoxifylline (Pentoxil/Trental), nandrolone decanoate (DecaDurabolin), oxandrolone (Oxandrin), and corticosteroids—in patients with AIDS, anorexia cachexia syndrome, and advanced cancer, only megestrol has been studied in malnourished elderly patients.

Two studies, mixed results

One placebo-controlled randomized clinical trial studied 45 malnourished patients who were recently discharged from an acute care hospital to a nursing home. The patients (predominately female, with a mean age of 83) were randomized into 4 treatment arms (placebo or megestrol 200 mg, 400 mg, or 800 mg daily) and followed for 63 days.

Only those receiving megestrol (400 mg or 800 mg daily) demonstrated a statistically significant increase in patient appetite and a dose-responsive increase in prealbumin level at the 20 day interim analysis (7.5 and 9.0 mg/dL, respectfully). But at the final assessment (63 days), only the 400-mg dose maintained a statistically significant increase in prealbumin over placebo. However, there was no significant improvement in serum albumin or clinical endpoints (weight, functional status, or health-related quality of life).1

In contrast, an earlier Veterans Administration (and predominantly male) study showed 13/21 of those treated with megestrol (800 mg daily for 12 weeks) noted weight gain (≥4 lb sustained at 3 months post-treatment), compared with 5/23 of those receiving placebo (number needed to treat [NNT]=2.5).2 Of note, only 9/26 patients had sustained weight gain in the megestrol group at the 12-month endpoint post-treatment, comparable with 7/25 in the placebo group.

Some small, but statistically significant, score improvements were noted during the treatment period in appetite and enjoyment of life; however, no differences were noted in scores on the more widely accepted Geriatric Depression Scale.

Adverse effects

As in all therapeutic interventions, benefit must be balanced against risk. The Megace ES package insert notes the following potential adverse effects: diarrhea, cardiomyopathy, palpitation, hepatomegaly, leukopenia, edema, paresthesia, confusion, convulsion, depression, neuropathy, hypesthesia and abnormal thinking, thrombophlebitis, pulmonary embolism, and glucose intolerance.3

To date, the prevalence rates of these potential adverse effects have only been studied in patients with AIDS. No data reflecting potential rates in elderly patients have been published.

Recommendations from others

The American Geriatric Society4 made 3 comments on appetite stimulation:

  1. There are no FDA-approved drugs available for the promotion of weight gain in older adults.
  2. A minority of patients receiving mirtazapine report appetite stimulation and weight gain.
  3. All drugs used for appetite have substantial potential adverse events.

We found only 1 national guideline on this topic: Unintentional Weight Loss in the Elderly from the University of Texas School of Nursing.5 The guideline indicates that drugs should not be used as first-line intervention in the elderly, as there has been inadequate testing in this population. Benefits are restricted to small weight gains without indication of decreased morbidity or mortality, improved quality of life, or improved functional ability.

Acknowledgments

The opinions and assertions contained herein are the private views of the author and not to be construed as official, or as reflecting the views of the US Air Force Medical Service or the US Air Force at large.

References

1. Reuben DB, Hirsch SH, Zhou K, Greendale GA. The effects of megestrol acetate suspension for elderly patients with reduced appetite after hospitalization: a phase II randomized clinical trial. J Am Geriatr Soc 2005;53:970-975.

2. Yeh SS, Wu SY, Lee TP, et al. Improvement in quality-of-life measures and stimulation of weight gain after treatment with megestrol acetate or suspension in geriatric cachexia: results of a double-blind, placebo controlled study. J Am Geriatr Soc 2001;48:485-492.

3. Megace Physicians’ Desk Reference 61st ed. Montvale, NJ: Thomson; 2007:2461-2463.

4. Malnutrition. Geriatrics at Your Fingertips [website]. Available at: www.geriatricsatyourfingertips.org/ebook/gayf_20.asp. Accessed August 6, 2007.

5. University of Texas, School of Nursing. Unintentional Weight Loss in the Elderly. Austin, Tex: University of Texas, School of Nursing; 2006. Available at: www.guideline.gov/summary/summary.aspx?doc_id=9435. Accessed August 6, 2007.

References

1. Reuben DB, Hirsch SH, Zhou K, Greendale GA. The effects of megestrol acetate suspension for elderly patients with reduced appetite after hospitalization: a phase II randomized clinical trial. J Am Geriatr Soc 2005;53:970-975.

2. Yeh SS, Wu SY, Lee TP, et al. Improvement in quality-of-life measures and stimulation of weight gain after treatment with megestrol acetate or suspension in geriatric cachexia: results of a double-blind, placebo controlled study. J Am Geriatr Soc 2001;48:485-492.

3. Megace Physicians’ Desk Reference 61st ed. Montvale, NJ: Thomson; 2007:2461-2463.

4. Malnutrition. Geriatrics at Your Fingertips [website]. Available at: www.geriatricsatyourfingertips.org/ebook/gayf_20.asp. Accessed August 6, 2007.

5. University of Texas, School of Nursing. Unintentional Weight Loss in the Elderly. Austin, Tex: University of Texas, School of Nursing; 2006. Available at: www.guideline.gov/summary/summary.aspx?doc_id=9435. Accessed August 6, 2007.

Issue
The Journal of Family Practice - 56(9)
Issue
The Journal of Family Practice - 56(9)
Page Number
761-762
Page Number
761-762
Publications
Publications
Topics
Article Type
Display Headline
Should we use appetite stimulants for malnourished elderly patients?
Display Headline
Should we use appetite stimulants for malnourished elderly patients?
Legacy Keywords
Megace; appetite; stimulant; weight; loss; unintended; elderly; aged; malnourished; Robert K. Persons DO; William Nichols MLS; Kayleen P. Papin MD
Legacy Keywords
Megace; appetite; stimulant; weight; loss; unintended; elderly; aged; malnourished; Robert K. Persons DO; William Nichols MLS; Kayleen P. Papin MD
Sections
PURLs Copyright

Evidence-based answers from the Family Physicians Inquiries Network

Disallow All Ads
Article PDF Media